You are on page 1of 61

C1 C3 1

A patient has developed congestive heart failure after experiencing his first myocardial infarction. The pulmonary signs and
symptoms the therapist expect to find include:
A. crackles and cough.
B. inspiratory wheezing and shortness of breath.
C. cough productive of thick yellow secretions.
D. crackles and clubbing of the digits.

C2 C8 1
A newborn who is 4 weeks premature is examined at birth using the APGAR test. Based on the following results, the
neonatal therapist suspects neurological complications are likely with an APGAR of:
A. 3 at 10 minutes.
B. 8 at 5 minutes.
C. 9 at 1 minute.
D. 8 at 1 minute.

C3 C7 3
A physical therapist requested that a PTA perform ultrasound on the left shoulder of a patient. During the treatment session,
the patient experienced an electrical shock. The physical therapist would not be responsible for any injury to the patient if
this was the result of:
A. faulty circuitry.
B. the patient touching the ultrasound machine.
C. failure of the PTA to use a ground fault interrupter.
D. failure of the PTA to use sufficient ultrasound gel.

C4 C6 3
A 55 year-old individual with documented coronary artery disease but no history of myocardial infarction is enrolled in an
exercise class that utilizes circuit-interval training. The MOST appropriate spacing of work-rest intervals to stress the
aerobic system is:
A. 1:1.5
B. 1:7
C. 1:10
D. 1:4.5

C5 C1 3
The BEST INITIAL intervention to improve functional mobility in an individual with a stable humeral neck fracture is:
A. pendulum exercises.
B. active resistive ROM.
C. I
D. sometrics for all shoulder musculature.
E. modalities to decrease pain.

C6 C1 3
A dancer with unilateral spondylolysis at L4 is referred for physical therapy. She complains of generalized low back pain
when she stands longer than one hour. Interventions for the subacute phase should include strengthening exercise for the:
A. multifidi working from full flexion back to neutral.
B. abdominals working from neutral to full flexion.
C. multifidi working from neutral to full extension.
D. abdominals working from full extension to full flexion.

C7 C1 1
A patient has fixed forefoot varus malalignment. Possible compensatory motion(s) or posture(s) might include:
A. excessive subtalar pronation.
B. genu recurvatum.
C. ipsilateral pelvic external rotation.
D. hallux varus.
C8 C7 3
The purpose of ABC Physical Therapy Center is to deliver excellent health care services in a unique, individualized and
participative manner at a reasonable cost. The provision of services under these conditions requires and encourages creative
involvement of physicians, staff, patients and their agents in all aspects of the care planning and delivery process. This is an
example of a:
A. mission statement.
B. values statement.
C. vision statement.
D. strategic plan.

C9 C5 3
A patient presents with weakness of the knee (2/5) resulting from an anterior cruciate ligament injury. The therapist’s
examination reveals moderate pain (5/10) and excessive translation of the tibia during active knee extension. The therapist
determines functional electrical stimulation (FES) is an appropriate intervention. The protocol for strengthening the
quadriceps and improving stability of the knee should consist of stimulation of the:
A. hamstrings immediately before the quadriceps to produce cocontraction.
B. quadriceps but not the hamstrings.
C. hamstrings but not the quadriceps.
D. quadriceps immediately before the hamstrings to produce cocontraction.

C10 C2 1
A therapist is reviewing a hospital record prior to examining a patient for the first time. The suspected diagnosis is multiple
sclerosis. On the neurologist’s note, the therapist finds the following: DTR right quadriceps is 2+, left quadriceps is 3+. The
therapist concludes that:
A. the right DTR is normal while the left is exaggerated.
B. both DTRs are abnormal and indicative of upper motor neuron syndrome.
C. the right DTR is exaggerated while the left is clearly abnormal.
D. both DTRs are abnormal and indicative of hyporeflexia.

C11 C3 1
A 72 year-old patient has an episode of syncope in the physical therapy clinic. The therapist attempts to rule out orthostatic
hypotension as the cause of her fainting. This is BEST done by:
A. checking resting BP and HR in sitting, then repeating measurements after standing for 1 minute.
B. checking HR and BP in supine after 5 minutes rest, then repeating in semi-Fowler position.
C. palpating the carotid arteries and taking HR; using the supine position for BP measurements.
D. checking HR and BP at rest, and after 3 and 5 minutes of cycle ergometry exercise.
C12 C4 3
An inpatient with a grade III diabetic foot ulcer is referred for physical therapy. Panafil has been applied to the necrotic
tissue BID. The wound has no foul smell; however, the therapist notes a green tinge on the dressing. In this case, the
therapist should:
A. document the finding and continue with treatment
B. document the finding and contact the physician immediately.
C. begin a trial of acetic acid to the wound.
D. fit the patient with a total contact cast.

C13 C8 1
The therapist receives a referral to examine the fall risk in an 82 year-old who lives alone and has had two recent falls. The
activity that represents the MOST common risk factor associated with falls in the elderly is:
A. turning around and sitting down in a chair.
B. climbing on a stepstool to reach overhead objects.
C. walking with a roller walker with hand brakes.
D. dressing while sitting on the edge of the bed.

C14 C6 1
A therapist is performing a prosthetic checkout on a patient with a transfemoral amputation. The prosthesis has been fitted
with a quadrilateral socket. A checkout of the walls of the socket should reveal that the:
A. anterior and lateral walls are 2 1/2 inches higher than the posterior and medial walls.
B. posterior and lateral walls are 2 inches higher than the medial and anterior walls.
C. height of the posterior wall is 2 inches less than all the other walls.
D. medial wall is 2 1/2 inches higher than the posterior wall while the anterior and lateral walls are the same
height.
C15 C7 3
A two month-old child with bilateral hip dislocations is being discharged home from an acute pediatric facility. The
therapist has developed a home exercise program and now needs to instruct her parents. The MOST important item for the
therapist to assess before instructing the parents is:
A. their degree of anxiety and attention.
B. the financial reimbursement plan.
C. their level of formal education.
D. the home environment.

C16 C7 3
The grip strength of a group of 50 to 60 year-olds was investigated. A mean score of 40, SD of 5, and range of 26-57 were
reported. The grip strength score for a given patient was determined to be 34. The therapist can safely conclude that in a
normal distribution this patient’s score fell within:
A. 95%.
B. 68%.
C. 75%.
D. 32%.

C17 C3 1
Which of the following clinical manifestations is NOT typical of early stage cystic fibrosis?
A. excessive appetite and weight gain.
B. increased pulmonary secretions with airway obstruction.
C. frequent recurrent respiratory infections.
D. salty skin and sweat.

C18 C8 1
An infant demonstrates that the ATNR is NOT obligatory when he/she can:
A. turn the head and bring the hand to mouth on the same side.
B. turn the head to either side and open the hand.
C. turn the head to one side and look at the extended arm on that side.
D. turn the head to one side and bring the opposite hand to mouth.

C19 C4 3
A 75 year-old patient is referred to physical therapy for back pain. Medical tests reveal he is at the end stage of pancreatic
cancer. The physician has told him he has cancer but has chosen not to inform the patient about the prognosis. If the patient
asks the therapist what his prognosis is, the therapist’s BEST response is to tell him:
A. to discuss his concerns with the physician.
B. to ask the nurse practitioner.
C. everything usually works out for the best.
D. the prognosis.

C20 C1 1
During surgery to remove an apical lung tumor, the long thoracic nerve was injured. Muscle weakness is 3+/5. It would be
best to implement exercises:
A. standing while performing wall push-ups.
B. standing using hand weights.
C. supine using weights.
D. supine using a pulley.

C21 C1 2
A physical therapist examination reveals: iliac crests high on the left; PSIS low and posterior on the left; ASIS high and
anterior on the left; standing flexion test shows that the left PSIS moves first and farthest superiorly; Gillet’s test
demonstrates the left PSIS moves inferiorly and laterally less than right; long sitting test shows the left malleolus moves
short to long; sitting flexion test is negative. In light of the above findings, the therapist’s diagnosis is:
A. left posterior rotated innominate.
B. left anterior rotated innominate.
C. left upslip.
D. iliac inflare on the left.
C22 C7 3
The highest level of provider risk in reimbursement is related to:
A. the capitation payment method.
B. the cost-based payment method.
C. the fee-for-service payment method.
D. the per diem payment method.

C23 C5 3
A patient presents with partial and full thickness burns on the chest and neck region. The therapist decides to apply TENS
prior to debridement to modulate pain. Which TENS mode should provide the BEST relief?
A. brief intense TENS.
B. conventional (high rate) TENS.
C. acupuncture-like (low rate) TENS.
D. modulated TENS.

C24 C2 3
A patient recovering from traumatic brain injury demonstrates instability during feeding while sitting in a wheelchair. The
therapist determines modification is necessary to ensure optimal function. The FIRST body segment or segments the
therapist would align is the:
A. pelvis.
B. head.
C. lower extremities.
D. trunk.

C25 C2 2
A patient demonstrates beginning recovery Stage 4 movements following a left CVA. The PNF pattern that represents the
BEST choice to promote continued recovery of the right upper extremity through the use of out-of-synergy movements is:
A. chop, reverse chop with right arm leading.
B. bilateral symmetrical D1 thrust and reverse thrust.
C. lift, reverse lift with right arm leading.
D. bilateral symmetrical D2F and D2E, elbows straight.

C26 C3 2
A 50 year-old individual has limited endurance as a result of a sedentary lifestyle. There is no history of cardiorespiratory
problems. Following an exercise tolerance test, which was negative, appropriate INITIAL exercise prescription parameters
for this individual would be:
A. 60 - 90% HRmax
B. 30 - 60% HRmax
C. 40 - 60% HRmax
D. 45 - 55% of VO2max

C27 C3 2
A patient is admitted to a coronary care unit with a mild myocardial infarction, After 2 days the patient is referred to
physical therapy for exercise conditioning. During an initial exercise session on the unit, he reports chest pain, appears
anxious, and wants to go back to bed to rest. The therapist’s BEST course of action is to terminate the exercise and:
A. have him sit down and continue to monitor his vital signs carefully.
B. monitor his vital signs and contact his doctor immediately.
C. assist the patient back to bed and contact the charge nurse on his floor.
D. assign the physical therapist assistant to assist him back to his bed and monitor his vital signs carefully.

C28 C4 3
A patient with a 10-year history of scleroderma is referred for physical therapy to improve functional status and endurance.
The patient was recently treated with corticosteroids for a bout of myositis. Examination findings reveal limited ROM and
fibrotic soft tissue along with hyperesthesia. The BEST choice for initial intervention is:
A. AROM exercises and walking in a therapeutic pool.
B. soft tissue mobilization and stretching.
C. closed chain and modified aerobic step exercises.
D. treadmill walking using body weight support at an intensity of 40% HRmax.
C29 C8 2
A patient is referred to physical therapy following a fall injury (fractured left hip with ORIF). Medical history reveals a
diagnosis of Stage I Alzheimer’s disease. At this stage, the behaviors the therapist would NOT expect to find are:
A. anxiety and irritability.
B. fragmented memory.
C. language comprehension problems.
D. restlessness and sundowning.

C30 C6 2
A patient recovering from stroke with minimal lower extremity weakness and spasticity is able to walk without an assistive
device. The therapist observes that as he walks he hikes his pelvis on the affected side during the swing phase. The BEST
initial intervention is:
A. bridging exercises progressing to sit-to-stand training.
B. marching while sitting on a therapy ball.
C. partial wall squats using a small ball held between the knees..
D. standing, marching with manual pressure applied downward on the pelvis.

C31 C7 3
The therapist is instructing a patient with traumatic brain injury how to lock the brakes on his wheelchair. He is right-
handed and his right upper extremity is more affected than his left. To obtain optimal results, the BEST training strategy is
to:
A. guide his right hand through the locking motions, then his left.
B. have him practice brake locking using his left hand to assist his right.
C. have him practice locking the brakes first with his left hand, and then his right.
D. verbally talk him through the locking motions practicing with both hands simultaneously.

C32 C7 3
A physical therapist wants to examine the effects of PNF using the technique of contract-relax on shoulder ROM. A group
of 10 patients with adhesive capsulitis were recruited. A matched group of patients were given straight plane active-assisted
exercise for the same length of time (3 times/week for 6 weeks). In this study the independent variable is:
A. active-assisted exercise.
B. adhesive capsulitis.
C. PNF contract-relax technique.
D. ROM.

C33 C3 1
A therapist is examining a patient with a 12-year history of emphysema. The clinical finding the therapist would NOT
expect to find is:
A. clubbing.
B. cor pulmonale.
C. cyanosis.
D. decreased A-P to lateral chest ratio.

C34 C1 3
A patient is receiving physical therapy following an ACL repair. After 4 weeks of treatment the patient still complains of
pain and instability even though he reports he has been faithful with his home exercise program and wearing his orthosis.
The original referral was for 7 to 8 weeks of physical therapy. The therapist’s BEST course of action is to:
A. complete the full 8 weeks of treatment and carefully document his lack of improvement to ensure full insurance
coverage.
B. discontinue the treatment; and discuss the situation with the patient and his referring physician.
C. recommend to the patient that he get a second medical opinion as the surgery seems unsuccessful.
D. re-examine the patient and implement a modified therapy program.

C35 C1 2
During a postural screen for chronic shoulder pain in a recreational swimmer, the therapist observes excessive internal
rotation of the shoulders and winging of the scapula during overhead motion. Intervention should focus on:
A. strengthening middle and lower trapezius and stretching of pectoral muscles.
B. strengthening of pectoral muscles and stretching of upper trapezius.
C. strengthening of rhomboids and stretching of upper trapezius.
D. strengthening of upper trapezius and stretching of pectoral muscles.
C36 C4 2
Re-examination of a patient with a dermal ulcer over the coccyx reveals a wound exposing the deep fascia. There is no
necrotic tissue, exudate is minimal, and the borders of the ulcer are diffusely covered with granulation tissue Previous
treatment has included daily whirlpool and wet-to-dry dressings with normal saline. Based on the re-examination,
intervention should consist of:
A. calcium alginate dressings
B. continuation of the same treatments.
C. whirlpool and hydrogel dressings
D. wound irrigation with pressures below 15 psi.

C37 C1 2
The spinal defect shown in the diagram should be managed with avoidance of lumbar spinal:

A. extension.
B. flexion.
C. lateral flexion.
D. rotation.

C38 C2 2
A computer specialist is unable to work because of weakness and altered sensation in her dominant right hand. She
complains of pain and tingling of the thumb, index finger, long finger, and radial half of the ring finger. The therapist
observes thenar weakness and atrophy. Strength, reflexes, and sensation are within normal limits throughout the remainder
of the right upper extremity. Her signs and symptoms are characteristic of:
A. carpal tunnel syndrome.
B. cervical root compression.
C. pronator teres syndrome.
D. ulnar nerve compression.

C39 C2 2
A patient presents with rapidly progressive symmetrical weakness that started in the distal lower extremity muscles but now
has ascended to include proximal trunk and upper extremity muscles. The motor segments of the lower cranial nerves are
also showing impairment. The patient complains of abnormal sensations of tingling and burning of the affected extremities.
Consciousness, cognition, and communication are all normal. These signs and symptoms are characteristic of:
A. amyotrophic lateral sclerosis.
B. Guillain-Barré syndrome.
C. multiple sclerosis.
D. post-polio syndrome.

C40 C3 3
A patient recovering from surgery for triple coronary artery bypass grafts is scheduled to begin a Phase III cardiac
rehabilitation program. During the resistance training portion of the circuit training program, the therapist instructs the
patient to AVOID the Valsalva maneuver because:
A. a cholinergic or vagal response can occur.
B. heart rate and blood pressure are likely to be elevated.
C. slowing of pulse and increased venous pressure are possible.
D. the decreased return of blood to the heart can lead to pitting edema.
C41 C4 2
A patient experiences color changes in the skin during position changes of the foot. During elevation, pallor develops.
When the limb is then positioned in the seated hanging position, hyperemia develops. These changes are indicative of:
A. arterial insufficiency.
B. chronic venous insufficiency.
C. deep vein thrombophlebitis.
D. lymphedema.

C42 C8 2
A 73 year-old is referred to physical therapy for an examination of balance. He has a recent history of falls (two in the last 6
months). Based on knowledge of balance changes in the elderly and scoring of standardized balance measures, the test data
that BEST indicates increased fall risk is:
A. Berg Balance score of 50.
B. Functional Reach of 7 inches.
C. Timed Get Up & Go test result of 13 seconds.
D. Tinetti Performance Oriented Mobility Assessment (POMA) score of 27.

C43 C6 3
A patient walks with a Trendelenburg gait. The MOST appropriate intervention to correct this problem is:
A. bridging, holding with Theraband around both thighs.
B. half kneeling, weight shifting onto the weak side (foot).
C. standing, stepping with the weaker limb, forward and backward.
D. supine, lateral leg slides.

C44 C3 2
A 72 year-old patient is walking on a treadmill in the physical therapy department while his vital signs are being monitored.
It is noted that his SaO2 drops from 97% to 95%. In this case, it would be BEST to:
A. not use supplemental O2.
B. place 2 liters of O2 by nasal cannula on the patient for the remainder of the exercise session.
C. place a 100% O2 face mask on the patient for the remainder of the exercise session.
D. place a 40% O2 face mask on the patient for the remainder of the exercise session.

C45 C8 2
The therapist is evaluating the needs of a 6 year-old child who is diagnosed with myelodysplasia at the T10 level. The
therapist determines the most appropriate mobility device for this child to use in the school environment is a:
A. bilateral HKAFO.
B. bilateral KAFO.
C. lightweight wheelchair.
D. parapodium.

C46 C1 3
A patient has limited right rotation caused by left thoracic facet joint capsular tightness. The intervention that would best
facilitate improved right rotation in sitting is:
A. trunk extension with left rotation.
B. trunk extension with right rotation.
C. trunk flexion with left rotation.
D. trunk flexion with right rotation.

C47 C1 1
EMG activity in the lower extremities during erect standing is continuous in the:
A. anterior tibialis and peroneals.
B. posterior tibialis and intrinsic foot muscles.
C. quadriceps femoris and anterior tibialis.
D. soleus and gastrocnemius.
C48 C7 3
A patient falls while walking in the parallel bars. The therapist is required to fill out an incident report of the event.
Information in the report should include the names of those involved and:
A. a description of the event and the corrective actions that will be taken.
B. the date of the occurrence, a description of the event, and the cause of the fall.
C. what occurred, when and where it occurred, and witness statements.
D. witness reports and therapist’s opinion as to the cause.

C49 C5 3
Four days ago, a patient sustained a deep contusion of the right lateral thigh as a result of a blow on the leg by a steel beam.
Following several cryotherapy treatments, the therapist decides to apply ultrasound. The parameters that are most
appropriate in this case are:
A. continuous US at 1 MHz.
B. continuous US at 3 MHz.
C. pulsed US at 1 MHz.
D. pulsed US at 3 MHz.

C50 C2 2
A 99 year-old woman was found unconscious at home. Two days later the therapist examines her in the hospital. Findings
include normal sensation and movement on the right side of the body with impaired sensation (touch, pressure,
proprioception) and paralysis on the left side of the body. The left side of her lower face and her trunk are similarly
impaired. The MOST LIKELY location of the lesion is the:
A. left parietal lobe.
B. left side of brainstem.
C. right parietal lobe.
D. spinal cord.

C51 C2 1
The loss of sensory function in peripheral neuropathy is often among the first noticeable symptoms. If more than one nerve
is involved, the sensory loss typically appears as:
A. allodynia of the feet accompanied by pronounced dorsiflexor weakness.
B. bandlike dysesthesias and paresthesias in the hips and thighs.
C. paresthesias affecting primarily the proximal limb segments and trunk.
D. stocking and glove distribution of the lower and upper extremities.

C52 C3 2
The therapist is supervising a Phase II cardiac rehabilitation class of 10 patients. One of the patients, who is being
monitored with radiotelemetry, is having difficulty. The therapist decides to terminate the patient’s exercise session and
reduce the exercise intensity upon observing:
A. 1 mm ST-segment depression, upsloping.
B. a 2° AV heart block.
C. an increase in HR 20 BPM above resting.
D. an increase in systolic BP to 150 and diastolic BP to 90.

C53 C4 3
A patient presents with severe claudication which is evident when he walks distances greater than 200 feet. He also exhibits
muscle fatigue and cramping of both calf muscles. Upon examination, the therapist finds his skin is pale and shiny with
some trophic nail changes. The BEST choice for intervention is to:
A. avoid any exercise stress until he has been on calcium channel blockers for at least 2 weeks.
B. begin with an interval walking program, exercising only to the point of pain.
C. utilize a walking program of moderate intensity, instructing the patient that some pain is expected and to be
tolerated.
D. utilize nonweightbearing exercises such as cycle ergometry.
C54 C8 3
A 72 year-old is hospitalized with diabetes and a large stage II plantar ulcer located over his right heel. He has been non-
weightbearing for the past 2 weeks as a result of the ulcer. The BEST intervention is:
A. clean and bandage with a sterile gauze dressing
B. clean and debride the wound, and apply a hydrogel dressing.
C. request a surgical consult as available wound dressings will not promote healing.
D. wash the foot and apply skin lubricants followed by a transparent film dressing.

C55 C6 3
The therapist is prescribing a wheelchair for a patient with left hemiplegia. The MOST appropriate feature to include in this
prescription is:
A. a 17.5 inch seat height.
B. a 20 inch seat height.
C. detachable arm rests
D. elevating legrests.

C56 C3 3
A patient who is participating in a cardiac rehab program suddenly collapses and falls to the floor. The physical therapist is
the lone rescuer on site. The therapist checks for a pulse and finds the patient unresponsive. The therapist calls the
emergency response number, and then opens the airway, checks breathing. The patient is not breathing, The therapist
should:
A. give 2 rescue breaths followed by 15 chest compressions, repeating the cycle.
B. give 2 rescue breaths followed by 30 chest compressions, repeating the cycle.
C. give chest compressions only at a rate of 100 per minute.
D. use the AED to shock the patient immediately; if unsuccessful then initiate CPR.

C57 C1 2
A patient who was casted for 3 weeks following a Grade III right ankle sprain has been referred to physical therapy for
mobility exercises. Examination shows a loss of 10 degrees of dorsiflexion. The patient will have the MOST difficulty in:
A. ambulating barefoot.
B. ambulating over rough surfaces.
C. descending a ramp.
D. descending stairs.

C58 C6 3
The torque output produced in the sitting position during isokinetic exercise involving the hamstrings is:
A. higher due to eccentric assistance of the quadriceps.
B. higher than the torque actually generated by the contracting hamstrings.
C. lower due to resistance of the quadriceps.
D. lower than the torque actually generated by the hamstrings.

C59 C1 3
A manual therapy technique utilized to correct a closing restriction of T5 on T6 is:
A. central P/A pressure at a 45 degree angle on the spinous process of T5 while stabilizing T6.
B. central P/A pressure at a 60 degree angle on the spinous process of T6 while stabilizing T5.
C. unilateral P/A pressure at a 45 degree angle on the right transverse process of T6 while stabilizing T5.
D. unilateral P/A pressure at a 60 degree angle on the left transverse process of T6 while stabilizing T5.

C60 C2 2
A patient presents with an acute onset of vertigo over night. Symptoms worsen with rapid change in head position. If the
head is held still, symptoms subside usually within 1 to 2 minutes. The MOST likely cause of this patient’s problem is:
A. acoustic neuroma.
B. benign paroxysmal positional vertigo.
C. bilateral vestibular neuritis.
D. Ménière’s disease.
C61 C3 2
A patient with a significant history for coronary artery disease tells the therapist he is currently taking atropine with careful
monitoring. Based on knowledge of the effects of this medication the therapist expects:
A. bradycardia at rest and with exercise.
B. increased heart rate and contractility at rest.
C. reduced blood pressure at rest and with exercise.
D. reduced myocardial ischemia and heart rate.

C62 C4 2
A 67 year-old patient recovering from stroke is on warfarin (Coumadin). During his rehabilitation, it would be important to
watch for:
A. cellulitis and xeroderma.
B. edema and dermatitis.
C. hematuria and ecchymosis.
D. palpitations and edema.

C63 C8 3
A 91 year-old patient has reduced vision as a result of bilateral cataracts. Which of the following is NOT an appropriate
intervention for this patient?
A. avoiding having him walk on shiny floor surfaces.
B. highlighting steps with pastel colors of blues and greens.
C. minimizing visual distractions in his immediate environment.
D. using high illumination for reading and ADLs.

C64 C6 2
A patient is having difficulty with stair climbing. During ascent, he is able to position his foot on the step above but is
unable to transfer the weight of his body up to the next stair level. The BEST intervention to solve this problem is:
A. hooklying, bridging.
B. plantigrade, knee flexion with hip extension.
C. standing, partial wall squats.
D. standing, side steps.

C65 C4 2
A 72 year-old patient has been hospitalized, on complete bedrest, for 10 days. A physical therapy referral requests
mobilization out-of-bed and ambulation. The patient complains that today his right calf is aching. If he gets up and moves
around he is sure he will feel better. The therapist’s examination reveals calf tenderness with slight swelling and warmth.
The therapist decides to:
A. ambulate the patient with his support stockings on.
B. begin with ankle pump exercises in bed and then ambulate.
C. postpone ambulation and report the findings immediately.
D. use only AROM exercises with the patient sitting at the edge of the bed.

C66 C8 1
A nine year-old boy with Duchenne’s muscular dystrophy is referred for homecare. The physical therapist should BEGIN
the examination by:
A. asking the child and his parents to describe the boy’s most serious functional limitations.
B. asking the parents to outline the boy’s past rehabilitation successes.
C. performing a complete motor examination.
D. performing a functional examination using the weeFIM.

C67 C1 1
During an examination of an adolescent female who complains of anterior knee pain, the therapist observes that the lower
extremity shows medial femoral torsion and toeing-in position of the feet. The lower extremity position may be indicative
of excessive hip:
A. anteversion.
B. lateral/external rotation.
C. medial/internal rotation.
D. retroversion.
C68 C1 3
A patient with osteoporosis and no fractures complains of increased mid and low back pain during breathing and other
functional activities. The MOST appropriate interventions for this patient include patient education and:
A. trunk extension and abdominal stabilization exercises.
B. trunk flexion and extension exercises.
C. trunk flexion, and rotation exercises.
D. trunk rotation and abdominal stabilization exercises.

C69 C1 2
A single, 22 year-old woman, who is 3 months pregnant, arrives at a therapist’s private practice complaining of shoulder
and leg pain. She has a black eye and some bruising at the wrists. The state in which the therapist practices has direct
access. An appropriate course of action is:
A. administer massage for bruising, TENs and ice modalities for pain, as indicated by the examination findings.
B. direct the patient to the nearest Ambulatory Care Center for physician evaluation.
C. do a comprehensive examination, and if the therapist suspects abuse report the findings to the appropriate
authorities.
D. perform a comprehensive examination and send her to the emergency room along with a copy of the findings.

C70 C5 3
A 73 year-old patient presents with a stage III decubitus ulcer on the plantar surface of the right foot. After a series of
conservative interventions with limited success, the therapist chooses to apply electrical stimulation for tissue repair. The
electrical current BEST suited in this case is:
A. high volt monophasic pulsed current.
B. low volt biphasic pulsed current.
C. medium frequency beat current.
D. medium frequency burst current.

C71 C5 3
A patient presents with pain radiating down the posterior hip and thigh as a result of a herniated disk in the lumbar spine.
The therapist decides to apply mechanical traction. If the patient can tolerate it, the PREFERRED patient position is:
A. prone with no pillow.
B. prone with pillow under the abdomen.
C. supine with both knees flexed.
D. supine with one knee flexed.

C72 C2 3
A patient recovering from stroke demonstrates hemiparesis of his right upper extremity and moderate flexion and extension
synergies (flexion stronger than extension). The therapist’s goal is strengthen the shoulder muscles first, specifically the
deltoid. The BEST choice is to promote:
A. abduction with elbow extension.
B. abduction with elbow flexion.
C. horizontal adduction with elbow extension.
D. horizontal adduction with elbow flexion.

C73 C2 2
A patient recovering from a middle cerebral artery stroke presents with gaze deviation of the eyes. In this type of stroke the
involved eye may deviate toward:
A. down and out.
B. the hemiplegic side.
C. the sound side.
D. up and in.

C74 C3 2
A patient is referred for physical therapy following an exercise tolerance test. The physician reports the test was positive
and had to be terminated at 7 minutes. Based on the therapist’s knowledge of this procedure, the therapist expects the
patient may have exhibited:
A. a hypertensive response with a BP of at least 170/95.
B. ECG changes from baseline of 1 mm ST-segment elevation.
C. increasing angina and dyspnea with progressive increases in the treadmill speed and grade.
D. ST segment depression from baseline of 3 mm horizontal or downsloping depression.
C75 C1 2
A college soccer player sustained a hyperextension knee injury when kicking the ball with his other lower extremity. The
patient was taken to the emergency room of a local hospital and was diagnosed with "knee sprain". He was sent to physical
therapy the next day for aggressive rehabilitation. As part of the examination to determine the type of treatment plan to
implement, the therapist conducts the test shown in the figure. The type of exercise that is indicated in the acute phase of
treatment if a positive test is found includes:

A. agility exercises.
B. closed chain terminal knee extension exercises.
C. open-chain terminal knee extension exercises.
D. plyometric functional exercises.

C76 C4 3
A patient with a grade III diabetic ulcer is being treated with a calcium alginate wound dressing. This type of dressing can
be expected to:
A. absorb exudate and allow rapid moisture evaporation.
B. facilitate autolytic debridement and absorb exudate.
C. provide semirigid support for the limb while maintaining a sterile field.
D. restrict bacteria from the wound while supporting the tissues.

C77 C4 3
The therapist receives a referral to treat a patient with a 5-year history of cirrhosis and Hepatitis B. The therapist should:
A. ask the patient to wear gloves and avoid contact.
B. avoid direct exposure to blood and body fluids.
C. use contact precautions.
D. use droplet transmission precautions.

C78 C6 2
A patient with a transtibial amputation is learning to walk using a PTB prosthesis. He is having difficulty maintaining
prosthetic stability from heel-strike to footflat. The muscles that are MOST LIKELY weak are the:
A. back extensors.
B. hip flexors.
C. knee extensors.
D. knee flexors.

C79 C3 2
A patient with a history of coronary artery disease and recent myocardial infarction is exercising in an inpatient Cardiac
Rehab Program. As the patient is new, continuous ECG telemetry monitoring is being done. The therapist observes the
following. The BEST course of action is to:

A. activate the emergency medical response team.


B. have the patient sit down and sent back to the room after a brief rest period..
C. have the patient sit down, continue monitoring, and notify the physician immediately.
D. have the patient sit down, rest, then resume the exercise at a lower intensity.
C80 C7 3
A patient recovering from traumatic brain injury is unable to bring the right foot up on the step during stair climbing
training. The BEST training activity is to:
A. passively bring the foot up and place it on the 7 inch step.
B. practice marching in place.
C. practice stair climbing inside the parallel bars using a 3 inch step.
D. strengthen the hip flexors using an isokinetic training device before attempting stair climbing.

C81 C3 3
Which of the following interventions is NOT beneficial for a patient with a right lower lobe viral pneumonia:
A. ambulation activity, monitoring SaO2, HR, RR, and BP.
B. breathing exercises, encouraging right lateral costal expansion.
C. postural drainage, percussion, and shaking to the right lower lobe.
D. teaching an independent exercise program to the patient.

C82 C4 2
A patient suffered a spinal cord injury with a complete injury (ASIA Level A) at T10. It is now three months post-injury
and she is refusing to participate in her functional training program because the major focus is wheelchair independence.
She is sure she is going to walk again. The therapist’s BEST approach is to:
A. discuss the harmful effects of denial and restrict all discussions to promoting wheelchair independence.
B. outline realistic short term goals to improve independence while maintaining for the possibility of further
recovery.
C. refer the patient for psychological counseling and discharge her from P.T.
D. send the patient home for a short time so she will recognize the need for wheelchair training.

C83 C8 3
The therapist is on a home visit, scheduled at lunchtime, visiting an 18 month-old child with moderate developmental
delay. The therapist notices the child and mother are experiencing difficulties with feeding. The child is slumped down in
the highchair and is unsuccessfully attempting to use a raking grasp to lift cereal pieces to her mouth. Both the child and
mother are frustrated. The FIRST intervention should be to:
A. recommend the mother feed the child baby food instead of cereal for a few more months.
B. recommend the mother return to breast feeding for a few more months.
C. reposition the child in a proper sitting position using postural supports.
D. work on desensitizing the gag reflex.

C84 C1 3
A patient is sent to physical therapy with a diagnosis of "frozen shoulder". The MOST effective mobilization technique for
restricted shoulder abduction is:
A. inferior glide at 55 degrees of abduction.
B. inferior glide at 95 degrees of abduction.
C. lateral glide in neutral position.
D. posterior glide at 10 degrees of abduction.

C85 C1 2
A patient presents with insidious onset of pain in the jaw that is referred to the head and neck regions. As best as he can
recall, it may be related to biting into something hard. Cervical ROM is limited in flexion by 20 degrees, cervical lateral
flexion limited to the left by 10 degrees. Mandibular depression is 10mm with deviation to the left, protrusion is 4mm, and
lateral deviation is 15mm to right and 6mm to left. Based on these findings the diagnosis for this patient would be:
A. capsule-ligamentous pattern of TMJ on the left.
B. cervical spine and TMJ capsular restrictions on the left.
C. weak lateral pterygoids on the left.
D. weak lateral pterygoids on the right.

C86 C7 3
A 35 year-old administrative assistant and mother of three is being treated for a Colles’ fracture. Her husband wants to look
at her medical record. The physical therapist should:
A. deny access to the chart unless written permission by his wife is granted.
B. give him the chart as he is a spouse and has a right to view the information.
C. let him look at the chart and be available to answer any questions.
D. not let him look at the chart because he may misinterpret the documentation.
C87 C3 2
A patient with a recent history of rib fractures suddenly becomes short of breath during secretion removal techniques. The
patient looks panicked and complains of sharp pain in the left chest. A quick screen shows a deviated trachea to the right
among other signs and symptoms. The MOST LIKELY explanation for the above is:
A. angina.
B. mucous plugging of an airway.
C. pneumothorax.
D. pulmonary emboli.

C88 C5 3
A patient presents with supraspinatus tendinitis. After the initial cryotherapy, the therapist decides to apply ultrasound. To
effectively treat the supraspinatus tendon, the therapist would place the shoulder joint in:
A. adduction and external rotation.
B. adduction and internal rotation.
C. slight abduction and external rotation.
D. slight abduction and internal rotation.

C89 C2 3
A patient is 5 weeks poststroke and is demonstrating good recovery of her right upper extremity, characterized as stage 4
recovery stage. The BEST choice for a training activity is to have the patient:
A. bear weight on the extended right arm in sitting.
B. put on socks and shoes while in sitting.
C. reach forward to bear weight with the right arm extended against the wall.
D. reach overhead with right arm straight.

C90 C2 2
A patient is 2 days post left CVA and has just been moved from the intensive care unit to a stroke unit. When beginning the
examination, the therapist finds the patient’s speech slow and hesitant. He is limited to one and two-word productions and
his expressions are awkward and arduous. However, he demonstrates good comprehension. His difficulties are consistent
with:
A. Broca’s aphasia.
B. dysarthria.
C. global aphasia.
D. Wernicke’s aphasia.

C91 C2 3
A patient recovering from traumatic brain injury is functioning at Stage IV on the Rancho Los Amigos Levels of Cognitive
Functioning Scale. During the therapist’s initial examination the patient becomes agitated and tries to bite the therapist. The
BEST course of action is to:
A. engage in a calming activity and document the behaviors.
B. postpone the examination for one week and then try again.
C. postpone the examination until later in the day when the patient calms down.
D. restructure the formal examination so the therapist can complete it in three very short sessions.

C92 C3 1
A therapist is working on a cardiac care unit in an acute care facility. After exercising a patient recovering from a
ventricular infarct, the therapist notices fatigue and dyspnea after mild activity. Later that day, on a return visit, the therapist
notices the patient has a persistent spasmodic cough while lying in bed. HR is rapid (140) and slight edema is evident in
both ankles. The patient appears anxious and agitated. The therapist suspects:
A. developing pericarditis.
B. impending myocardial infarction.
C. left ventricular failure.
D. right ventricular failure.
C93 C3 2
A marathon runner is examined in physical therapy for anterior pain in the right lower leg. Her resting heart rate is found to
be 46 bpm. The MOST LIKELY explanation for this is that:
A. a compensatory response to prolonged endurance activity is depressed heart rate with decreased stroke volume.
B. a low heart rate is suggestive of a hypotensive disorder.
C. incipient coronary pathology is often characterized by an abnormally low heart rate.
D. prolonged endurance training has resulted in a low heart rate.

C94 C4 2
A patient has been taking corticosteroids (hydrocortisone) for management of adrenocortical insufficiency. She is referred
to physical therapy for mobility training following a prolonged hospitalization. Potential adverse effects that one can expect
from prolonged use of this medication include:
A. atrophy and osteoporosis.
B. confusion and depression.
C. decreased appetite and weight loss.
D. hypotension and myopathy.

C95 C4 1
A therapist is treating a patient with deep partial-thickness burns over 35% of the body (chest and arms). Wound cultures
reveal a bacterial count in excess of 105 per gram of tissue on the anterior left arm The therapist can reasonably expect:
A. the burn area is pain free as all nerve endings in the dermal tissue were destroyed.
B. the infected wound can convert the area to a full-thickness burn.
C. the risk of hypertrophic and keloid scars is low as there is no viable tissue.
D. with antibiotics spontaneous healing can be expected.

C96 C6 3
Recently, a 10 year-old patient has begun walking with supination of her foot. With her shoe off, the therapist finds a new
callus on the lateral side of the metatarsal head of the 5th toe. The BEST choice for orthotic prescription is:
A. scaphoid pad.
B. Thomas heel.
C. viscoelastic shoe insert with forefoot lateral wedge.
D. viscoelastic shoe insert with forefoot medial wedge.

C97 C6 3
A 74 year-old patient is recovering from a right total hip replacement (posterolateral incision, cementless fixation). The
MOST appropriate type of bed-to-wheelchair transfer to teach is to have the patient use a:
A. lateral slide transfer using a transfer board.
B. squat- pivot transfer to the surgical side.
C. stand-pivot transfer to the sound side.
D. stand-pivot transfer to the surgical side.

C98 C6 3
A patient with paraplegia at the T10 level wants to participate in wheelchair basketball. He asks the therapist what options
he should look for in a wheelchair. The therapist tells him it would be important to include:
A. a folding frame.
B. a mid-scapular seat back.
C. a rigid frame.
D. hard-rubber tires.

C99 C8 3
The therapist is treating a child with mild developmental delay secondary to 7 weeks prematurity at birth. The child is now
8 months old and is just learning to sit. The BEST choice for training activity is:
A. prone tilting reactions.
B. sideward protective extension in sitting.
C. standing tilting reactions.
D. supine tilting reactions.
C100 C3 2
A 80 year-old patient with emphysema and a history of hypertension performs a 12 minute walking exercise tolerance test.
He was able to walk 1,106 feet. His vital signs prior to exercise were: HR 104, BP 130/76, SaO 2 93%. At peak exercise his
vital signs were: HR 137, BP 162/74, SaO2 92%. To calculate his exercise intensity parameters, the BEST method to use is:
A. 40 to 50% of Max METs.
B. 40 to 85% HR reserve (Karvonen’s formula).
C. 70 to 80% of HRmax
D. 70 to 85% of age adjusted predicted HRmax

C101 C8 2
An expected outcome for a fifteen year-old boy with Duchenne’s muscular dystrophy is:
A. dependent wheelchair mobility.
B. independent in ambulation with Lofstrand crutches.
C. independent in ambulation with no assistive devices
D. independent in wheelchair mobility.

C102 C1 1
When performing scoliosis screening in a school setting, the optimal age to screen for girls is:
A. 15-17.
B. 6-8.
C. 9-11.
D. 12-14.

C103 C1 3
A patient with a confirmed left C6 nerve root compression due to foraminal encroachment complains of pain in his left
thumb and index finger. The MOST effective cervical position to alleviate this radicular pain in weightbearing is:
A. left sidebending.
B. lower cervical extension.
C. lower cervical flexion.
D. right rotation.

C104 C1 1
A patient is standing with excessive subtalar pronation. Possible correlated motions or postures are:
A. tibial and femoral external rotation, with pelvic external rotation.
B. tibial and femoral internal rotation with pelvic external rotation.
C. tibial, femoral, and pelvic external rotation.
D. tibial, femoral, and pelvic internal rotation.

C105 C7 3
When performing a chart audit, the therapist realizes a date of service was documented inappropriately. The therapist
should:
A. erase the incorrect date; then make and date the correction.
B. put a single line through the incorrect date, initial, then make and date the correction.
C. use "white out", initial; then add the correct date of service.
D. write over the date with a different color ink, then make and initial the correction.

C106 C5 3
A therapist is applying high volt pulsed current to the vastus medialis to improve patellar tracking during knee extension.
The patient complains that the current is uncomfortable. To make the current more tolerable to the patient, yet maintain a
good therapeutic effect, the therapist should consider adjusting the:
A. current intensity.
B. current polarity.
C. pulse duration.
D. pulse rate.
C107 C2 2
A patient has a 10-year history of multiple sclerosis and presents with drooping of the right upper eyelid, constriction of the
pupil, and vasodilation with absence of sweating on the face and neck. These signs are characteristic of:
A. Argyll Robertson pupil.
B. homonymous hemianopsia.
C. Horner’s syndrome.
D. nystagmus.

C108 C2 2
A patient presents with symptoms of uncoordinated eye movements and profound gait and trunk ataxia. He has difficulty
with postural orientation to vertical and tends to tip over even if his eyes are open. Examination of the extremities reveals
little change in tone or coordination. The therapist suspect involvement of the:
A. basal ganglia.
B. premotor cortex.
C. spinocerebellum.
D. vestibulocerebellum.

C109 C3 1
A patient with a 10-year history of diabetes complains of cramping, pain, and fatigue of the right buttock after walking 400
feet or climbing stairs. When he stops exercising, the pain goes away immediately. The skin of the involved leg is cool and
pale. The therapist checks his medical record and finds no mention of this problem. The therapist suspects:
A. peripheral arterial disease.
B. peripheral nerve injury.
C. Raynaud’s phenomenon.
D. spinal root impingement.

C110 C4 2
A patient with low back pain has marked elevation of blood pressure. He complains of mild to severe midabdominal pain
that increases upon exertion. Palpation reveals a pulsing mass in the lower abdomen. The therapist should:
A. discontinue treatment and notify his physician immediately.
B. instruct in relaxation exercises as a pulsating mass is not unusual with hypertension.
C. instruct the patient to contact his physician at the conclusion of therapy.
D. provide hot packs to the abdomen to help relieve the muscle spasm.

C111 C4 3
A patient with a 12-year history of diabetes and a small, purulent tunneling wound located on the left heel is referred for
wound lavage. The therapist’s BEST choice is to irrigate the wound using:
A. a syringe with Dakin’s solution while the patient is in the whirlpool.
B. hydrogen peroxide spray.
C. whirlpool with povidone-iodine.
D. whirlpool with water temperature at 20°C .

C112 C8 1
An 85 year-old wheelchair dependent resident of a community nursing home has a diagnosis of organic brain syndrome,
Alzheimer’s type, Stage 2. During the therapist’s initial interview, the patient demonstrates limited interaction, mild
agitation, and keeps trying to wheel her chair down the hall. As it is late in the day, the therapist decides to resume the
examination the next morning. The patient is most likely exhibiting:
A. disorientation to time and date.
B. frustration because of an inability to communicate.
C. inattention as a result of short term memory loss.
D. sundowning behavior

C113 C6 3
To increase the stride length of a patient with a right transfemoral amputation who uses a total contact prosthesis, the
therapist should:
A. facilitate the gluteals with tapping over the muscle belly.
B. provide anterior directed resistance to the right PSIS during swing.
C. provide posterior directed resistance to the left ASIS during swing.
D. provide posterior directed resistance to the right ASIS during stance.
C114 C6 3
A patient with a complete C7 spinal cord injury is having difficulty with pushups while in his wheelchair. The MOST
appropriate lead-up activity to enhance wheelchair pushups is:
A. prone-on-elbow pushups.
B. shoulder shrugs.
C. supine bench press using 50% one repetition max.
D. supine-on-elbows pushups.

C115 C7 3
A therapist is concerned that a student PT, who is on a final clinical rotation, is having difficulty interacting with the
patients. Specifically, the student does not seem to be willing to listen or demonstrate tolerance and sensitivity to patient
needs. The MOST appropriate conclusion the therapist can reach is that affective objectives for the clinical education
experience are not being met. The primary deficit is:
A. Level 1.0 Receiving.
B. Level 3.0 Valuing.
C. Level 4.0 Organization.
D. Level 5.0 Adherence to a professional code of ethics.

C116 C7 3
A therapist wants to investigate the effectiveness of use of the therapeutic pool for decreasing pain in a group of patients
with fibromyalgia. Two groups of patients were recruited. One group was assigned to exercises and walking in the pool 3
times/week for 6 months. The other group was assigned to a gym walking program for the same amount of time. At the end
of the study, outcomes were assessed using the McGill Pain Questionnaire and the Health Status Questionnaire. In order to
improve reliability, the lead investigator should:
A. have another therapist reassess after 6 months and compare to normalized scores.
B. have the same therapist reassess the patients after 6 months.
C. perform all the final assessments himself and compare to the initial assessments performed by a core group of
therapists.
D. utilize a core of 4 experienced therapists to randomly complete all the assessments.

C117 C3 2
An apparently healthy individual has many risk factors for coronary artery disease. He is interested in improving his overall
fitness and cardiac health. Following an exercise tolerance test, which was asymptomatic, he is referred for an exercise
class. The most accurate measure of exercise intensity to monitor during his first exercise session is:
A. heart rate.
B. MET level.
C. rating of perceived exertion (RPE).
D. respiratory rate.

C118 C8 2
A mother brings her 8 week-old infant to be examined at Early Intervention because she noticed that the infant was taking
steps in supported standing at two weeks but was not able to do it now. The therapist should:
A. explain that this is normal and that the stepping was a newborn reflex that has gone away.
B. explain this was due to a stepping reflex that will re-emerge again around 10 months.
C. recommend that a full developmental exam be performed by the Early Intervention team.
D. recommend that the mother bring the infant to a pediatric neurologist.

C119 C1 1
A patient complains of pain with mouth opening that makes it difficult for her to eat foods that require chewing.
Examination revealed active mouth opening to be within normal limits of:
A. 15-24mm.
B. 35-44mm.
C. 50-64mm.
D. 65-74mm.
C120 C1 3
Correction of flexible forefoot varus with excessive subtalar pronation is accomplished by a customized orthosis with:
A. lateral forefoot posting.
B. medial forefoot and rearfoot varus posting.
C. medial forefoot posting.
D. rearfoot varus posting.

C121 C1 2
A patient presents with complaints of tingling and paresthesias in the median nerve distribution of the right forearm and
hand. The following tests were found negative bilaterally: Adson, hyperabduction, costoclavicular, Phalen’s, and the ulnar
nerve Tinel sign. Based on this information, the diagnosis that has NOT been ruled out is:
A. carpal tunnel syndrome.
B. pronator teres syndrome.
C. thoracic outlet syndrome.
D. ulnar nerve entrapment.

C122 C7 3
A patient scheduled for a 30-minute treatment session arrives 10 minutes late. The subsequent patients are also scheduled
for 30 minutes sessions and there is no break in the therapist’s schedule to accommodate for the patient’s tardiness. The
therapist should:
A. ask the patient if there is a more convenient time for the appointment in order to complete a 30-minute session.
B. send the patient home until the next scheduled appointment, with a request to be punctual.
C. treat the patient and bill for 30 minutes of scheduled treatment time.
D. treat the patient, and bill for the 20-minute session given.

C123 C5 3
A patient has been referred to the therapist s/p fracture of the femur six months ago. The cast was removed, but the patient
is unable to volitionally contract the quadriceps. The therapist decides to apply electrical stimulation to stimulate the
strengthening of the quadriceps muscle. The BEST choice of electrode size and placement is:
A. large electrodes, closely spaced.
B. large electrodes, widely spaced.
C. small electrodes, closely spaced.
D. small electrodes, widely spaced.

C124 C2 3
A 65 year-old patient is recovering from a right CVA. Due to a series of medical complications she is still bedridden 6 days
post-stroke. The therapist wants to reduce the expected negative effects of developing spasticity. The BEST choice of bed
position for this patient is:
A. sidelying on the affected side, with the affected shoulder positioned directly underneath, the hip slightly extended
with knee flexed on a pillow.
B. sidelying on the sound side, affected arm and leg extended at the side, with a pillow between the knees.
C. supine, trunk in midline with small pillow under the scapula, arm extended on supporting pillow, and a small towel
roll under the knee.
D. supine, trunk in slight lateral flexion to the sound side with elbow flexed and supported on a pillow, leg straight.

C125 C2 1
If the subject’s vision is blocked either by having the subject close the eyes or by placing a barrier between the part being
tested and the subject’s eyes, the therapist can effectively examine:
A. conscious proprioception but not discriminative touch.
B. discriminative touch and fast pain but not proprioception.
C. somatosensory integrity.
D. vestibular/visual/somatosensory integration.

C126 C3 2
A therapist is examining a patient in the coronary intensive care unit. On auscultation the therapist hears an adventitious S3
heart sound. This finding is indicative of:
A. aortic valve dysfunction.
B. congestive heart failure.
C. pericarditis.
D. pulmonary valve dysfunction.
C127 C3 2
A therapist is asked to advise a healthy 67 year-old individual who wants to take part in a graduated conditioning program
by joining the "Mall Walkers Club". The therapist’s BEST approach to prescribing the intensity of exercise for this
individual is:
A. 4-8 MET level walking.
B. 70-85% of maximal age-related HR
C. dyspnea scale.
D. HR reserve formula and Ratings of Perceived Exertion.

C128 C4 2
A 72 year-old patient recovering from stroke has been using a bilateral exerciser (UBE) to strengthen muscles in his
affected right upper extremity. He is now experiencing burning pain in his shoulder that worsens when his limb is touched
or moved. He also presents with paresthesias and pitting edema in the dorsum of the hand. ROM of the wrist and fingers is
painful and diminished. The therapist’s BEST course of action is to:
A. discontinue exercise and use ice for pain relief.
B. discontinue UBE exercise, splint the hand and wrist until pain and swelling disappear.
C. discontinue UBE exercise; use massage and active assistive ROM.
D. switch to interval exercise and lower the resistance on the UBE.

C129 C8 2
A 74 year-old patient has had two recent falls coming home from Bingo after dark. Her outside steps are well lit. She’s
unsure why she has fallen but tells the therapist both times she fell just as she came into her house, before she even had a
chance to put her purse down and turn on the inside lights. The therapist suspect a problem with:
A. decreased corneal sensitivity.
B. decreased ocular scanning movements.
C. loss of accommodation and near vision.
D. poor light adaptation.

C130 C6 3
A patient presents with 2/5 muscle strength in both lower extremities and 3/5 strength in the upper extremities. The MOST
appropriate transfer to teach this patient to move from bed to wheelchair is:
A. dependent 1-man squat transfer.
B. sliding board.
C. stand pivot.
D. stand-by assist.

C131 C6 3
A patient with a spinal cord injury at the level of T1 is in the community phase of his mobility training. In order for him to
navigate a standard height curb with his wheelchair, the therapist tell him to:
A. ascend backwards with the large wheels first.
B. descend backward with the trunk upright and arms hooked around the push handles.
C. lift the front casters and ascend in a wheelie position.
D. place the front casters down first during descent.

C132 C3 2
A patient with a long history of systemic steroid use for asthma control has a contraindication for percussion if there is
evidence of:
A. barrel chest.
B. BP > 140/90.
C. decreased bone density.
D. intercostals muscle wasting.
C133 C1 2
A patient presents with a rapid onset of severe weakness of all small muscles of the hands, sharp pleuritic pain in the
shoulder and subscapular area, and a hoarse voice for the past three weeks. She is a hair stylist, has a smoking history of 20
years, is not on any medications, and has not been ill. Her referral states examine and treat. Based on the above information
this patient is MOST LIKELY exhibiting symptoms of:
A. a Pancoast tumor.
B. thoracic outlet syndrome
C. ascending Guillain-Barré syndrome.
D. a C5-6 bilateral foraminal stenosis.

C134 C1 2
An examination of a patient reveals the following shoulder signs and symptoms: excessive AROM and PROM; pain with
activity, and on palpation; normal resisted isometric contractions; a positive load/shift test; and negative X-ray findings.
The MOST LIKELY diagnosis is:
A. atraumatic shoulder instability.
B. impingement.
C. rotator cuff lesion.
D. traumatic anterior shoulder dislocation.

C135 C5 3
The therapist is applying cervical traction using a cervical harness. The patient complains of pain in the temporomandibular
joint during the treatment. The therapist should consider:
A. decreasing the treatment time.
B. discontinuing traction.
C. readjusting the harness and continuing with the treatment.
D. reducing the traction poundage and continuing with the treatment.

C136 C6 2
A therapist is treating a patient with a diagnosis of right shoulder rotator cuff tendonitis. The findings of a worksite
ergonomic assessment indicate that the worker is required to perform repetitive reaching activities above shoulder height.
The most appropriate worksite modification would be to:
A. allow the worker to take more frequent rests to avoid overuse.
B. provide the worker with a standing desk for daily activities.
C. provide the worker with a taller, sit-stand chair.
D. reposition the height of the shelf and items to below shoulder height.

C137 C2 3
A patient has been diagnosed with impingement syndrome of the shoulder. Following a course of modalities to control pain
and inflammation, progression is to an exercise program to restore normal function of the shoulder. The BEST PNF
diagonal pattern to improve function of the shoulder is:
A. D1 extension.
B. D1 flexion.
C. D2 extension.
D. D2 flexion.

C138 C6 2
A patient presents with pain of the right Achilles tendon as well as on the plantar aspect of the right heel. Pain developed
insidiously and has now lasted several months. On gait analysis the therapist observes abnormal supination throughout the
stance phase of gait. The BEST choice for orthotic intervention is a:
A. cushion heel with a rearfoot valgus post.
B. flexible shoe insert with forefoot varus post.
C. metatarsal pad.
D. UCBL insert.
C139 C3 2
A patient has a 10 year history of peripheral vascular disease affecting the right lower extremity. During auscultation of the
popliteal artery the therapist would NOT expect to find:
A. 1+ pulses.
B. 2+ pulses.
C. a bruit.
D. absence of detectable blood flow.

C140 C4 2
A therapist is working in a major medical center and is new to the acute care setting. An orientation session for new
employees concerns infection control. The therapist learns that the most common infection transmitted to healthcare
workers is:
A. hepatitis A.
B. hepatitis B.
C. HIV.
D. tuberculosis.

C141 C4 2
A patient presents with pain, joint swelling, subcutaneous olecranon nodules, and increased erythrocyte sedimentation rate.
These findings are characteristic of:
A. fibromyalgia.
B. osteoarthritis.
C. rheumatoid arthritis.
D. systemic lupus erythematosus.

C142 C6 1
A patient with a transfemoral amputation and an above-knee prosthesis demonstrates knee instability while standing. His
knee buckles easily when he shifts his weight. The therapist suspect the cause of his problem is a:
A. prosthetic knee set too far anterior to the TKA line.
B. prosthetic knee set too far posterior to the TKA line.
C. tight extension aid.
D. weak gluteus medius.

C143 C6 3
A 17 year-old individual with developmental disabilities is referred to a wheelchair clinic for a new wheelchair. She
presents with a severe kyphoscoliosis. The therapist determines the BEST wheelchair modification to order is a:
A. contoured foam seat.
B. firm seat back with lateral posture supports and increased seat depth.
C. firm seat with lateral knee positioners.
D. sling seat with dense foam cushion.

C144 C6 3
A patient is referred for orthotic gait training after receiving a reciprocating gait orthosis. In order for this patient to walk
correctly, it is important to instruct her in the correct sequence. She should shift her weight:
A. onto her crutches and swing both legs through together to a position in front of her crutches.
B. onto her crutches and swing one leg, then the other forward.
C. onto her walker and one leg, tuck her pelvis by extending the upper trunk, and swing her other leg through.
D. onto her walker, extend the upper trunk, and swing both legs forward together to approach the walker.

C145 C7 3
A patient who is undergoing spinal cord rehabilitation is viewed as uncooperative by staff. He refuses to complete the
training activities outlined for him to promote independent functional mobility. A review of his history reveals that
previously he was the director of his own company, with a staff of 20. The MOST appropriate strategy the therapist can
adopt is to:
A. carefully structure the activities and slow down the pace of training.
B. have him work with a supervisor since he works best with people in authority.
C. involve him in goal setting and have him participate in structuring the training session.
D. refer him to another therapist who is male in the hopes that he will have better luck in engaging the patient.
C146 C7 3
A group of 10 patients is recruited into a study investigating the effects of relaxation training on blood pressure. One group
of patients is scheduled to participate in a cardiac rehabilitation program which includes relaxation training 3 times a week
for 12 weeks. The other group of patients is instructed to perform activities as usual. At the conclusion of the study there
was no significant difference between the groups; BP decreased significantly in both groups. The investigator can
reasonably conclude:
A. both groups had blood pressures initially so high that reductions should have been expected.
B. cardiac rehabilitation is not effective in reducing blood pressure.
C. the activities of the non-rehab group were not properly monitored and may account for these results.
D. the rehab group was not properly monitored.

C147 C6 2
A patient is using a right KAFO. During orthotic checkout, the therapist discover the height of the medial upright is
excessive. As she transfers weight to the orthotic leg during gait, the therapist expects that this patient will demonstrate:
A. anterior trunk bending.
B. lateral lean toward the left.
C. lateral lean toward the right.
D. posterior trunk bending.

C148 C6 2
During gait, a patient with hemiparesis drags his toes during swing. Upon further examination, he has weak dorsiflexors
(able to lift the foot against gravity through 1/2 range) and a grade of 2 upon examining tone in his plantar flexors using the
Modified Ashworth Scale. An appropriate orthotic modification to correct this problem is:
A. a dorsiflexion assist.
B. a dorsiflexion stop.
C. a solid ankle AFO.
D. spiral AFO.

C149 C8 2
A therapist is examining a 24 month-old child and observes that the child can sit independently, creep in quadruped, pull-
to-stand, cruise sideways, but not walk without support. The therapist concludes that this child is exhibiting:
A. delay in achieving developmental milestones.
B. normal cephalocaudal motor development.
C. normal gross motor development.
D. slow maturation that is within normal limits.

C150 C1 3
Therapist hand/finger placements for posterior to anterior (PA) mobilization techniques to improve down-gliding/closure of
the T7-8 facet joints should be located at the:
A. spinous process of T6.
B. spinous process of T8.
C. transverse processes of T7.
D. transverse processes of T8.

C151 C1 2
A patient complains of waking up several times at night from severe "pins and needles" in both hands. On awakening, her
hands feel numb for half an hour, and she complains of clumsiness with fine hand movements. The therapist’s examination
revealed paresthesias in the medial forearm and hypothenar region; reduced grip and pinch strength; and normal tendon
reflexes. Based on the above examination findings the MOST appropriate diagnosis is:
A. carpal tunnel syndrome.
B. pronator teres syndrome.
C. thoracic outlet syndrome.
D. ulnar nerve entrapment.
C152 C5 2
An auto mechanic is referred for physical therapy with a diagnosis of degenerative joint disease affecting C2 and C3. The
patient complains of pain and stiffness in the cervical region and transient dizziness with some cervical motions. The
MOST appropriate INITIAL examination procedure is:
A. a vertebral artery test.
B. Adson’s maneuver.
C. Lhermitte’s test.
D. Oppenheim test.

C153 C2 3
A 22 year-old otherwise healthy patient is recovering from a complete spinal cord injury at the level of L2. Functional
expectations for this patient include:
A. ambulation using bilateral AFOs and canes
B. ambulation using bilateral KAFOs and a reciprocating walker.
C. ambulation using bilateral KAFOs, crutches and a swing-through gait.
D. wheelchair locomotion using an active duty lightweight chair.

C154 C2 1
To examine a patient with a suspected deficit in graphesthesia, the therapist would ask the patient, with eyes shut, to
identify:
A. a series of letters traced on the hand.
B. different objects placed in the hand and manipulated.
C. different weighted, identically shaped cylinders placed in the hand.
D. the vibrations of a tuning fork when placed on a bony prominence.

C155 C2 3
A patient recovering from stroke is ambulatory without an assistive device. He demonstrates a consistent problem with an
elevated and retracted pelvis on the affected side. The BEST therapeutic exercise strategy is to manually apply:
A. anterior directed pressure during swing.
B. downward compression during stance.
C. light resistance to forward pelvic rotation during swing.
D. light resistance to posterior pelvic elevation during swing.

C156 C2 2
A therapist suspects lower brainstem involvement in a patient with amyotrophic lateral sclerosis. Examination findings
reveal motor impairments of the tongue with ipsilateral wasting and deviation on protrusion. These findings confirm
involvement of cranial nerve:
A. IX.
B. X.
C. XI.
D. XII.

C157 C3 2
A patient is in the intensive care unit following myocardial infarction. Upon examination of the ECG tracings in the
medical record the therapist observes the following changes: ST elevation and T wave inversion in leads II, III, and AVF.
The probable location of the infarct is the:
A. inferior wall.
B. lateral wall.
C. posterior wall.
D. anterior wall.

C158 C4 1
A patient is hospitalized in the ICU with extensive trauma following a motor vehicle accident. A review of her medical
record reveals the following lab values: hematocrit 28%, hemoglobin 10 g/100ml, and serum WBC 12,000/mm3. The
MOST appropriate conclusion the therapist can reach is:
A. all values are abnormal.
B. hematocrit and hemoglobin values are abnormal; WBC is normal.
C. only hematocrit values are abnormal.
D. only serum WBC is abnormal.
C159 C4 2
A patient with a transfemoral amputation is unable to wear his total contact prosthesis for the past 4 days. Examination of
the residual limb reveals erythema and edema extending over most of the lower anterior limb. He tells the therapist his limb
is very itchy and painful after he scratches it. The MOST LIKELY cause of his symptoms is:
A. cellulitis.
B. dermatitis.
C. herpes zoster.
D. impetigo.

C160 C4 3
A patient is largely confined to bed and has a stage IV sacral pressure ulcer of three months duration. The BEST choice of
intervention is:
A. a two-inch, convoluted foam mattress.
B. gentle wound cleansing and wet-to-dry gauze dressings.
C. nutritional supplements and pressure relief with a flotation mattress.
D. surgical repair.

C161 C6 2
Following a hip fracture that is now healed, a patient presents with weak hip flexors (2/5). All other muscles are within
functional limits. During gait, the therapist expects that the patient may walk with:
A. a circumducted gait..
B. backward trunk lean.
C. excessive hip flexion.
D. forward trunk lean.

C162 C6 3
A 72 year-old patient with a left transfemoral amputation complains that his left foot is cramping and when he sits it feels
all twisted under him. The therapist’s BEST choice of intervention is:
A. appropriate bed positioning with the residual limb in extension.
B. hot packs and continuous ultrasound to the residual limb.
C. icing and massage to the residual limb.
D. iontophoresis to the distal residual limb using hyaluronidase.

C163 C7 3
A researcher uses a group of volunteers (healthy, college students) to study the effects of Swiss ball exercises on ankle
ROM and balance scores. Twenty volunteers participated in the 20 minute ball exercise class 3 times a week for 6 weeks.
Measurements were taken at the beginning and end of the sessions. Significant differences were found in both sets of scores
and reported at the local PT meeting. Based on this research design, the therapist concludes:
A. Swiss ball exercises are an effective intervention to improve ankle stability following chronic ankle sprain.
B. the Hawthorne effect may have influenced the outcomes of the study.
C. the reliability of the study was threatened with the introduction of systematic error of measurement.
D. the validity of the study was threatened with the introduction of sampling bias.

C164 C3 3
A patient has a very large right-sided bacterial pneumonia. Her oxygen level is dangerously low. The body position that
would MOST LIKELY improve her PaO2 is:
A. left sidelying with the head of the bed in the flat position.
B. prone-lying with the head of the bed in the Trendelenburg position.
C. right sidelying with the head of the bed in the flat position.
D. supine-lying with the head of the bed in the Trendelenburg position.

C165 C1 1
An examination of a 46 year-old woman reveals drooping of the shoulder, rotatory winging of the scapula, an inability to
shrug the shoulder, and complaints of aching in the shoulder. Based on these findings, the cause of these symptoms would
MOST LIKELY be due to:
A. a lesion of spinal accessory nerve.
B. a lesion of the long thoracic nerve.
C. muscle imbalance.
D. strain of the serratus anterior.

C166 C5 3
A patient with spastic hemiplegia is referred to the therapist for ambulation training. The patient is having difficulty with
standing-up from a seated position due to cocontraction of the quadriceps and hamstrings during the knee and hip extension
phase. The therapist wishes to use biofeedback beginning with simple knee extension exercise in the seated position. The
plan is to progress to sit-to-stand training. The initial biofeedback protocol should consist of:
A. high detection sensitivity with recording electrodes placed closely together.
B. high detection sensitivity with recording electrodes placed far apart.
C. low detection sensitivity with recording electrodes placed closely together.
D. low detection sensitivity with recording electrodes placed far apart.

C167 C2 2
Examination of a patient recovering from stroke reveals a loss of pain and temperature sensation on the left side of the face
along with loss of pain and temperature sensation on the right side of the body. All other sensations are normal. The
therapist suspect a lesion in the:
A. left cerebral cortex or internal capsule.
B. left posterolateral medulla or pons.
C. midbrain.
D. right cerebral cortex or internal capsule.

C168 C2 2
A patient is taking the drug Baclofen to control spasticity following spinal cord injury. This medication can be expected to
decrease muscle tone and pain. Adverse reactions of concern to the physical therapist can include:
A. drowsiness and muscle weakness.
B. headache with visual auras.
C. hypertension and palpitations.
D. urinary retention and discomfort.

C169 C2 2
Symptoms of dysdiadochokinesia, dysmetria, and action tremor can be expected with a lesion located in the:
A. neocerebellum.
B. spinocerebellum.
C. vermis.
D. vestibulocerebellum.

C170 C3 2
A patient is exercising in a Phase 3 outpatient cardiac rehabilitation program that utilizes circuit training. One of the
stations utilizes weights. The patient lifts a 5 lb weight, holds it for 20 seconds and then lowers it slowly. The therapist
corrects the activity and tells the patient to reduce the length of the static hold. The static exercise can be expected to
produce:
A. abnormal oxygen uptake.
B. higher heart rate and arterial blood pressure.
C. lower heart rate and arterial blood pressure.
D. reduced normal venous return to the heart and elevated blood pressure.

C171 C4 3
A ten year-old boy with hemophilia fell and injured himself while skateboarding. He was admitted to a pediatric acute care
facility and a therapist sees him that afternoon. Examination reveals a hemarthrosis in his left knee. The BEST initial
intervention for this patient is:
A. a hot pack for the knee and instruction in nonweightbearing exercises.
B. a pool program to maintain ROM and strength while he is nonweightbearing.
C. ice, elevation and a splint for the limb.
D. instruct the patient in crutch use to protect the joint and assist in early return to walking.
C172 C8 2
A 72 year-old male patient recovering from a fractured hip repaired with ORIF has recently been discharged home. During
a home visit, his wife tells the therapist he woke up yesterday morning and told her he couldn’t remember much. Upon
examination, the therapist finds some mild motor loss in his right hand and anomia. The therapist affirms the presence of
short-term memory loss. The BEST course of action is to:
A. advise the family to document and record any new problems they notice over the next week, then report back to
the therapist.
B. ignore the findings as they are expected following surgical anesthesia.
C. refer him to his physician as the therapist suspects a small stroke.
D. refer him to his physician as the therapist suspects Alzheimer’s dementia.

C173 C6 3
A 16 year-old with a 4-year history of Type I diabetes is insulin dependent. He wants to participate in cross country
running. The physical therapist working with the school team advises the athlete to measure his plasma glucose
concentrations before, and after running. In addition, the student should:
A. avoid carbohydrate-rich snacks within 12 hours of the race.
B. consume a carbohydrate after the race to avoid hyperglycemia.
C. consume a carbohydrate before or during the race to avoid hypoglycemia.
D. increase insulin dosage immediately before running.

C174 C6 3
A 61 year-old patient with a transtibial amputation has a short residual limb. The BEST choice for prosthetic replacement is
a:
A. supracondylar suspension.
B. supracondylar/suprapatellar suspension.
C. Syme’s suspension.
D. thigh corset.

C175 C6 1
A physical therapist assistant is ambulating a patient using a three-point crutch gait. The patient is unsteady and fearful of
falling. The patient does not appear to understand the correct gait sequence. The supervising therapist’s BEST strategy is to:
A. instruct the PTA to have the patient sit down and utilize mental practice of the task.
B. instruct the PTA to use a distributed practice schedule to ensure patient success.
C. intervene and teach the correct sequence since the PTA is apparently unable to deal with this special situation.
D. tell the PTA and patient to stop the ambulation and work on dynamic balance activities instead.

C176 C1 1
The therapist is reviewing x-rays from a patient with a Grade 2 spondylolisthesis. The BEST radiographic view to observe
this bony anomaly is:
A. anteroposterior.
B. lateral.
C. oblique.
D. posteroanterior.

C177 C5 3
An athlete presents with pain and muscle spasm of the upper back (C7-T8) extending to the lateral border of the scapula.
This encompasses a 10 x 10 cm area on both sides of the spine. If the ultrasound unit only has a 5 cm 2 soundhead, the
therapist should treat:
A. each side allotting five minutes for each section.
B. each side allotting two and a half minutes for each section.
C. the entire area in five minutes.
D. the entire area in ten minutes.
C178 C5 2
A patient with chronic cervical pain is referred to an outpatient physical therapy clinic. Past medical history reveals:
appendectomy, 12 years ago; chronic heart disease; demand-type pacemaker, 8 years ago; whiplash injury, 2 years ago.
Presently the patient complains of pain and muscle spasm in the cervical region. The modality that should not be considered
in the case is:
A. hot pack.
B. mechanical traction.
C. transcutaneous electrical stimulation.
D. ultrasound.

C179 C2 2
In posturography, patients who sway more or fall under conditions with the eyes closed and platform moving (condition 5)
or with the visual surround moving and platform moving (condition 6) are likely to demonstrate:
A. problems with sensory selection.
B. somatosensory dependency.
C. vestibular deficiency.
D. visual dependency.

C180 C2 2
A patient presents with severe, frequent seizures originating in the medial temporal lobes. Following bilateral surgical
removal of these areas, he is unable to remember any new information just prior to the surgery to the present. He cannot
recall text he read minutes ago and cannot remember people he has met repeatedly. These outcomes are indicative of:
A. a primary deficit from the loss of the amygdala.
B. loss of integration of the temporal lobe with the basal ganglia and frontal cortex.
C. loss of procedural memory and integration with frontal cortex.
D. loss of the hippocampus and declarative memory function.

C181 C3 2
A 14 year-old with a body mass index of 30kg/m2 and a history limited participation in physical activities is referred for
exercise training. The nutritionist has prescribed a diet limiting his caloric intake. The BEST initial exercise prescription is:
A. 2 daily sessions of 30 minutes at 40-70% VO2max
B. 2 weekly, sessions of 60 minutes at 50% VO2max
C. 3 weekly sessions of 50 minutes at 70-85% VO2max
D. 3 weekly, sessions of 30 minutes at 60-70% VO2max

C182 C3 2
While on a home visit, a 9 month-old infant becomes unresponsive. The therapist tell the mother to call for emergency
medical services. The therapist’s IMMEDIATE next step is to tilt the infant’s head back and give 2 full breaths, while
covering both the mouth and nose. Chest compressions should then be given. The correct procedure for chest compressions
for the infant is to:
A. compress with one hand to a depth of one third to one half the depth of the chest at a rate of 100/min.
B. compress lightly with 2 fingers to a depth of 2 inches at a rate of 60/min.
C. compress with both hands to a depth of 1 inch at a rate of 60/min.
D. compress lightly with 2 fingers to a depth of 1/2 to 1 inch at a rate of 100/min.

C183 C4 2
A patient in an exercise class develops muscle weakness and fatigue. Examination reveals leg cramps and hyporeflexia. He
also experiences frequent episodes of postural hypotension and dizziness. Abnormalities on his ECG include a flat T wave,
prolonged QT interval, and depressed ST segment. The therapist suspects:
A. hyperkalemia
B. hypocalcemia
C. hypokalemia
D. hyponatremia
C184 C8 2
A 64 year-old patient with advanced coronary artery disease and diabetes is receiving functional mobility training in a P.T.
clinic. While walking after lunch, the patient experiences difficulty breathing, starts hyperventilating, and suffers an
episode of syncope. The MOST LIKELY cause of this problem is:
A. coronary artery disease.
B. hyperglycemia.
C. postprandial hypertension.
D. seizures.

C185 C6 3
The MOST appropriate position to guard a patient who is descending stairs for the first time using crutches and non-
weightbearing on the right is to:
A. stand behind and slightly to the right side.
B. stand behind and slightly to the right side.
C. stand in front and slightly to the left side.
D. stand in front and slightly to the right side.

C186 C6 3
A patient with paraplegia at the T10 level is being discharged home. The rehab team is assisting the patient’s wife in
modifying the home to be barrier-free. A new first floor bedroom and bathroom wing are being built. The builder asks the
therapist about the entrance. The therapist recommends:
A. a ramp with a grade of 12% and a level landing at top of at least 50 by 50 inches.
B. a ramp with a slope of 1:12 with a level landing at the top of at least 60 by 60 inches.
C. door width of 36 inches with an outside door that opens out.
D. door widths of at least 40 inches with a handrail 34 inches high.

C187 C2 3
A patient has a 10 year history of Parkinson’s disease and has been on levodopa for the past 6 years. He has fallen 3 times
in the past month resulting in a Colles’ fracture. The therapist decides to try postural biofeedback training using a platform
balance training device. The training sequence should focus on:
A. decreasing the limits of stability and improving anterior weight displacement.
B. decreasing the limits of stability and improving posterior weight displacement.
C. increasing the limits of stability and improving anterior weight displacement.
D. increasing the limits of stability and improving center of pressure alignment.

C188 C3 2
A patient is recovering from myocardial infarction and is referred for supervised exercise training. While working out on a
treadmill, the patient begins to develop mild shortness of breath. Upon inspection of the ECG readout the therapist
determines:

A. sinus rhythm with downsloping ST segment depression.


B. sinus rhythm with SA blocks.
C. sinus rhythm with upsloping ST segment depression.
D. tachycardia with abnormal P waves.
C189 C8 3
A 16 year-old boy with Duchenne’s muscular dystrophy has been confined to using a power wheelchair for the past three
years and is beginning to develop a 10 degree Cobb angle scoliosis. The BEST strategy to help slow this spinal curvature at
this time would be to:
A. alternate the side of the wheelchair power control.
B. emphasize spinal extension exercises.
C. emphasize spinal rotation exercises.
D. order a new wheelchair with a reclining seat back.

C190 C1 2
A patient presents with a complaint of severe neck and shoulder pain of two days duration. The patient reports falling
asleep on the couch watching TV, and has been stiff and sore since. There is tenderness of the cervical muscles on the right,
with increased pain upon palpation. PROM is most limited in flexion, then sidebending left, and then rotation left, and
active extension. Sidebending right and rotation right is also painful. Based on these examination findings, the patient’s
diagnosis is:
A. cervical radiculopathy.
B. cervical strain.
C. facet syndrome.
D. herniated disc.

C191 C6 2
A postal worker (mail sorter) complains of numbness and tingling in his right hand in the median nerve distribution. When
the therapist evaluates his work tasks, the therapist notes that he is required to key in the zip codes of about 58 letters per
minute. An appropriate administrative control to decrease his exposure would be to:
A. provide him with a height adjustable chair to position his wrists and hands in a neutral alignment.
B. provide the worker with a resting splint to support his wrist.
C. require the worker to attend a cumulative trauma disorder educational class.
D. use job rotation during the workday.

C192 C5 3
A patient with a traumatic brain injury presents with hemiparesis of the left upper extremity. The examination reveals slight
cutaneous and proprioceptive impairment, fair (3/5) strength of the shoulder muscles and triceps and slight spasticity of the
biceps. Voluntary control of the patient’s left arm has not progressed since admission. The therapist decides to use
functional electrical stimulation and place the active electrode on the triceps to facilitate active extension of the elbow. The
timing sequence BEST to apply is:
A. 2-second ramp up, 10-second stimulation, no ramp down.
B. 2-second ramp up, 5-second stimulation, 2-second ramp down.
C. 5-second ramp up, 5-second stimulation, 5-second ramp down.
D. no ramp up, 10-second stimulation, 2-second ramp down.

C193 C2 2
Following a traumatic brain injury, a patient presents with significant difficulties in learning new skills. He is wheelchair
dependent and will need to learn how to transfer (a skill he has never done before). The BEST strategy to enhance his
motor learning is to:
A. provide bandwidth feedback using a random practice schedule.
B. provide consistent feedback using a blocked practice schedule.
C. provide summed feedback after every few trials using a serial practice schedule.
D. use only guided movement to ensure correct performance.

C194 C2 2
A patient recovering from a CVA presents with predominant involvement of the contralateral lower extremity and lesser
involvement of the contralateral upper extremity. The patient also demonstrates mild apraxia. These clinical manifestations
are characteristic of:
A. anterior cerebral artery syndrome.
B. basilar artery syndrome.
C. middle cerebral artery syndrome.
D. posterior cerebral artery syndrome.
C195 C3 1
A patient presents with significant intermittent claudication with onset after two minutes of walking. On further
examination, the therapist would expect to find:
A. bright red appearance of the extremity in both gravity dependent and independent positions.
B. elevation-induced pallor and dependent redness with the extremity in the gravity dependent position.
C. grayish-white appearance of the extremity in both gravity dependent and independent positions.
D. little or no changes in color with changes in extremity position.

C196 C4 3
Following her cesarean section, a patient tells the therapist that she is anxious to return to her pre-pregnancy level of
physical activity (working out at the gym 3 days a week and running 5 miles every other day). The therapist’s BEST advice
is to tell her to resume activities with:
A. a walking program progressing to running after 5 weeks.
B. abdominal crunches with return to running after 1 month.
C. pelvic floor and gentle abdominal exercises for the first 4-6 weeks.
D. pelvic floor exercises and refrain from all other exercise and running for at least 6-8 weeks.

C197 C8 2
A 77 year-old patient lives alone and is referred for home physical therapy services to improve functional mobility. He
refuses to get out of his chair. Upon examination, he appears irritable, with poor concentration and memory. He tells the
therapist he has not been sleeping well and has no energy or desire to do anything. He appears anorexic. The MOST
LIKELY explanation for his symptoms is:
A. alcohol abuse.
B. depression.
C. hypothyroidism.
D. pseudodementia.

C198 C6 3
The rehabilitation team is completing a home visit to recommend environmental modifications for a 72 year-old patient
who is scheduled to be discharged next week. He is wheelchair dependent. The bathroom has not been adapted. Which of
the following recommendations is NOT appropriate?
A. adding a tub seat.
B. adding horizontal grab bars positioned at 45 inches.
C. raising the toilet seat to 18 inches.
D. taking the door off to widen the entrance to 32 inches.

C199 C6 2
During gait, the lateral pelvic tilt on the side of the swing leg, observed during frontal plane analysis, serves to:
A. control forward and backward rotations of the pelvis.
B. reduce knee flexion at mid stance.
C. reduce peak rise of the pelvis.
D. reduce physiological valgum at the knee.

C200 C1 2
A patient experiences central thoracic pain while sitting which increases during the day. There are no complaints in the
night or morning. Lifting and carrying heavy objects increase the pain. Based on the above information the MOST LIKELY
diagnosis would be:
A. herniated nucleus pulposus of the spine.
B. spondylodiscitis.
C. thoracic postural syndrome.
D. traumatic compression fracture.
C1 C3 I
Crackles and cough.
Patients who present with a myocardial infarction and congestive heart failure have changes to their pulmonary exam, the
most common being crackles and dry cough. Inspiratory wheezing occurs with extreme airway narrowing, which is not a
hallmark of congestive heart failure. The cough associated with congestive heart failure is most likely nonproductive.
Clubbing of the digits is a sign of chronic hypoxia. In this case scenario, this patient presents with his first myocardial
infarction. It is not possible for any chronic changes to have taken place at this point.

C2 C8 I
3 at 10 minutes
The Apgar Score is based on heart rate, respiration, muscle tone, reflex irritability (grimace) and color (appearance). Apgar
scores are routinely assigned at 1 and 5 minutes and occasionally at 10 min. post birth. Scores of 8-10 at 1 minute post
birth are normal; scores between 0-3 at 1 and 5 minutes are extremely low and indicative of the need for resuscitation.
Neurological complications are likely with extremely low Apgar scores, particularly at 10 minutes.

C3 C7 III
Faulty circuitry.
The physical therapist, in this case, correctly delegated the ultrasound treatment to the PTA. Every individual (PT, PTA) is
liable for their own negligence; however supervisors may assume liability of workers if they provide faulty supervision or
inappropriate delegation of responsibilities (not evident in this case). Faulty circuitry is the correct answer. Physical
therapists are not liable for defective equipment unless they contributed to its malfunction, e.g. dropping the ultrasound
machine and continuing to use it in treatment without having it checked. The institution assumes liability if the patient was
harmed as a result of an environmental problem. The standard of practice is such that a ground fault interrupter and ade-
quate ultrasound gel are used during administration of ultrasound.

C4 C6 III
1:1:5
A rest interval equal to one and a half times the work interval allows exercise to begin before recovery is complete, thus
effectively stressing the aerobic system. The range of effective work-recovery ratio for aerobic training is 1:1 to 1:5.

C5 C1 III
Pendulum exercises
This individual will typically be immobilized with a sling for a period of 6 weeks. After one week the sling should be
removed to have the patient perform the pendulum exercise to prevent shoulder stiffness. Resistive exercises are not
indicated during this early period. Heat modalities may be effective in reducing pain but do not improve mobility.

C6 C1 III
Multifidi working from full flexion back to neutral.
Performing strengthening exercises to the multifidi from flexion to neutral will not stress the pars defect. Abdominal
strengthening will not provide the segmental stability needed with this condition. Lumbar extension beyond neutral and
rotation will tend to aggravate the condition in the early stages of rehabilitation.

C7 C1 I
Excessive subtalar pronation.
Possible compensatory motions or postures for forefoot varus malalignment include: plantarflexed first ray, hallux valgus,
excessive midtarsal or subtalar pronation or prolonged pronation; excessive tibial, tibial and femoral, tibial, femoral and
pelvic internal rotation, and/or all the contralateral lumbar spine rotation.

C8 C7 II
Mission statement
There are four fundamental documents that would provide guidance and direction to all members of the organization. They
are the values statement, vision statement, mission statement, and the strategic plan. A mission statement is a fundamental
document that gives direction to all members of an organization as they carry out their current duties and plan for the
future. It describes the organization's purpose; what the organization produces, sells, who the customers are, and to some
degree, how people are treated by members of the organization. The values statement is the organization's proclamation
regarding what behaviors are right and wrong, or acceptable or not. The vision statement is inspirational to look forward to
what can be achieved and to enlist support and efforts to make the envisioned possibilities realities. Strategic plans identify
strategies used to meet the overall organizational mission.
C9 C5 III
Hamstrings immediately before the quadriceps to produce cocontraction.
Stimulating the hamstrings just prior to stimulating the quadriceps will stabilize the tibia and prevent anterior tibial
translation during knee extension and during cocontraction of both muscles. This would prevent the moving tibia from
placing tension on the injured anterior cruciate ligament.

C10 C2 I
The right DTR is normal while the left is exaggerated.
Deep tendon reflexes are graded on a 1-4 scale. A 2+ score for DTRs is normal. Abnormal scores include: 0 (absent); 1+
decreased; 3+ exaggerated or hyperactive; and 4+ clonic. Scores of 3+ and 4+ are indicative of spastic hypertonia.

C11 C3 I
Checking resting BP and HR in sitting, then repeating measurements after standing for 1 minute.
Orthostatic hypotension is a fall in BP with elevation of position, i.e., from supine to sitting or sitting to standing. A small
increase or no increase in heart rate upon standing may suggest baroreflex impairment. An exaggerated increase in HR
upon standing may indicate volume depletion.

C12 C4 III
Document the finding and continue with treatment.
A greenish tinge to the dressing is expected with the use of Panafil. Panafil is a keratolytic enzyme used for selective
debridement. A greenish or yellowish exudate can be expected. If the exudate was green and had a foul smell,
Pseudomonas aeruginosa should be suspected and acetic acid would be the topical agent of choice. A total contact cast can
only be used once the wound is free of necrotic tissue.

C13 C8 I
Turning around and sitting down in a chair
Most falls occur during normal daily activity. Getting up or down from a bed or chair, turning, bending, walking,
climbing/descending stairs all are high risk activities. Only a small percent of individuals fall during clearly hazardous
activities (e.g. climbing the stepstool). Proper use of an assistive device reduces the risk of falls.

C14 C6 I
Anterior and lateral walls are 2 ½ inches higher than the posterior and medial walls.
The anterior and lateral walls are built 2 ½ to 3 inches higher than the posterior and medial walls to ensure proper
positioning on the ischial seat.

C15 C7 III
Their degree of anxiety and attention.
A needs assessment should include a determination of the level of anxiety and ability to attend to the instructions given.
While the other factors may also be considered, they do not represent immediate priorities for hospital-based instruction.

C16 C7 III
95%
In a normal distribution (bell-shaped curve), 95% of scores can be expected to fall within +2 or -2 SD of the mean; 99% of
scores fall within +3 or-3 SD of the mean.

C17 C3 I
Excessive appetite and weight gain.
Cystic fibrosis is an inherited disorder affecting the exocrine glands of the hepatic, digestive, respiratory systems. The
patient with CF is prone to chronic bacterial airway infections and progressive loss of pulmonary function from progressive
obstructive lung disease. Early clinical manifestations include an inability to gain weight despite excessive appetite and
adequate caloric intake. Excessive weight gain is rarely a finding in a patient with cystic fibrosis. All the other choices are
typically present along with persistent coughing, wheezing, and reduced exercise tolerance.

C18 C8 I
Turn the head and bring the hand to mouth on the same side.
ATNR causes extension of upper extremity on the side the head is turned toward. Bringing the hand to the mouth would not
be possible with an obligatory reflex.
C19 C4 III
To discuss concerns with the physician.
The therapist should respond empathetically and use the moment to talk to the patient about his illness. However the
prognosis should not be discussed as that is the domain of the physician.

C20 C1 I
Standing while performing wall push-ups.
The long thoracic nerve supplies the serratus anterior muscle. With a muscle grade of 3+/5, the patient can then begin
functional strengthening using standing wall push ups using resistance provided by the patient’s own body. The other
exercises would not be optimal for strengthening a Fair plus serratus anterior.

C21 C1 II
Left posterior rotated innominate.
A posterior rotated innominate is a unilateral iliosacral dysfunction. The question outlines positive physical .findings, both
static and dynamic, found with this dysfunction. One of these positive findings alone does not confirm the diagnosis of left
rotated posterior innominate.

C22 C7 III
The capitation payment method.
Capitation payment methodology allows the managed care insurer to shift some of the financial risk of care delivery to the
selected health care provider. The provider agrees to accept a set fee in return for the delivery of services to a specified
group of managed care enrollees. Under the cost-based and fee-for-service payment method the level of risk is low. The
payment risk factors are: charges set below costs, costs exceed caps, and non-payment of co-payments and deductibles.
Under the per diem payment method the level of risk is medium. The payment risk factors are: per diem rate set below
costs, services included under the per diem rate are not specifically defined, high potential for variation in case required,
length of service too short, providers inability to control daily service utilization. Under the capitation payment method the
risk is high. The payment risk factors include: capitation rate is set below costs, service utilization projections are lower
than actual utilization, providers lack complete control over the delivery of services, and providers inability to control serv-
ice utilization.

C23 C5 III
Brief intense TENS.
Brief intense TENS is used to provide rapid onset, but short-term relief during painful procedures. The pulse rate and pulse
duration are similar to conventional TENS, however the current intensity is increased to the patient’s tolerance.

C24 C2 III
Pelvis.
Modification of the pelvic position in a neutral posture promotes good lumbar and trunk alignment. Many postural prob-
lems are correctable by aligning the pelvis first and achieving a stable base.

C25 C2 II
Chop, reverse chop with right arm leading.
Both chop and reverse chop move the affected arm out-of-synergy. Thrust is an out-of-synergy pattern while reverse thrust
is in-synergy. Lift is an out-of-synergy pattern while reverse lift is in-synergy. The same is true for bilateral symmetrical
D2F (out-of-synergy) and D2E (in-synergy).

C26 C3 II
60-90% HRmax
An appropriate initial exercise prescription for an asymptomatic individual is 60-90% of HR max, which is equivalent to 50-
85% of V02max or 50-85% of heart rate reserve (Karvonen formula).

C27 C3 II
Have him sit down and continue to monitor his vital signs carefully.
If the chest pain (angina) is exercise induced, this is an indication to terminate the exercise session (myocardial demand is
exceeding myocardial oxygen supply). Recovery is expected after a period of rest. If the patient is still anxious after the
rest, it is reasonable to return the patient to his room and inform the nurse. This should be done by the therapist personally
in order to carefully monitor his status. The PTA should not be expected to evaluate his chest pain or reach a determination
about its significance. This is not an emergency situation.
C28 C4 III
AROM exercises and walking in a therapeutic pool.
Scleroderma (progressive systemic sclerosis) is a chronic, diffuse disease of connective tissues causing fibrosis of skin,
joints, blood vessels and internal organs. Patients typically demonstrate symmetric skin thickening and visceral
involvement of the GI tract, lungs, heart and kidney along with hypersensitivity to touch. The best choice for initial
intervention is to exercise in the pool. The warmth and buoyancy of the water will enhance the patient’s movements and
decrease pain. The other choices are too aggressive at this time and risk increasing the patient’s pain, thereby limiting any
benefits in flexibility and endurance.

C29 C8 II
Language comprehension problems.
Early Stage I Alzheimer’s disease is characterized by memory loss, absentmindedness, difficulty concentrating and
occasional word-finding problems. Profound communication deficits (inability to speak or eat), global deterioration of
mental functions (delusions, hallucinations, fragmented memory), agitation, and pacing (sundowning) are all characteristic
of late stages of this disease.

C30 C6 II
Marching while sitting on a therapy ball.
Hip hiking is a compensatory response for weak hip and knee flexors or extensor spasticity. Active exercises for the hip and
knee flexors (marching) is the most appropriate intervention. Downward manual pressure on the pelvis strengthens hip
hikers. The other choices focus on strengthening hip and knee extensors.

C31 C7 III
Have him practice locking the brakes first with his left hand, and then his right.
Transfer training is appropriate to use with this patient. Practice is performed with less affected extremity first and then
progressed to practice with the more affected extremity. Guided movement (manual or verbal) represents a less active
approach than the transfer of training approach. The more passive the performance, the slower the learning. Bilateral tasks
are more difficult than performing the task with one limb.

C32 C7 III
PNF contract-relax technique
The independent variable is the activity or factor believed to bring about a change in the dependent variable. The type of
exercise being investigated is PNF contract-relax technique. The dependent variable is the difference in behavior that
results as a result of the intervention (independent variable); in this case, ROM.

C33 C3 I
Decreased AP to lateral chest ratio.
The barreled chest commonly seen in emphysema is measured as an increased A-P to lateral ratio. Cor pulmonale is
hypertrophy or failure of the right ventricle and results from disorders of the lungs, pulmonary vessels, or chest wall.
Clubbing (bulbous, shiny fingertips and toes) and cyanosis (bluish or grayish skin) are both present with chronic lung
disease.

C34 C1 III
Discontinue the treatment; and discuss the situation with the patient and his referring physician.
According to the Guide for Professional Conduct, "when physical therapists judge that an individual will not benefit from
their services, they shall so inform the individual receiving the services. Physical therapists shall avoid over-utilization of
their services". Consultation with his referring physician is indicated. After 4 weeks, this patient is in pain and complains of
instability. The PT intervention does not seem to be of benefit. Discontinuing treatment without informing the physician or
patient would be inappropriate.

C35 C1 II
Strengthening the middle and lower trapezius and stretching of pectoral muscles.
Abnormal posture that produces excessive internal rotation of the shoulders may result in chronic shoulder impingement
syndrome due to a loss of scapular stability with overhead motion. Shoulder pain is likely to continue until a balance
between anterior and posterior trunk musculature is achieved. The anterior chest muscles (pectorals) are shortened and need
stretching and posterior trunk muscles (middle and lower trapezius) are stretched and need strengthening.
C36 C4 II
Wound irrigation with pressures below 15 psi.
If the ulcer is clean, whirlpool can damage incipient granulation tissue and should be discontinued. Wet-to-dry dressings
help remove necrotic tissue. Calcium alginate is used in the presence of heavy exudates, which is not the case here.
Hydrogel would be best as it is non-adherent, keeps wound moist and protects granulation buds; however, whirlpool is
inappropriate. Low pressure wound irrigation helps to decrease colonization and prevent infection.

C37 C1 II
Extension
With spondylolisthesis there is typically an anterior slippage of one vertebra on the vertebra below. Because of the anterior
shearing forces acting at the vertebra caused by the wedge shape of the vertebra and gravity, spinal extension positions
should be avoided.

C38 C2 II
Carpal tunnel syndrome
The pattern of motor and sensory loss corresponds to the median nerve distribution in the hand. The most likely cause is
carpal tunnel syndrome. Pronator teres syndrome (also a median nerve problem) produces similar deficits along with
involvement of the flexors of the wrist and fingers. Cervical root compression would also produce proximal deficits in
strength and sensation.

C39 C2 II
Guillain-Barré syndrome
These signs and symptoms are characteristic of Guillain-Barre syndrome, a peripheral neuropathy in which there is
inflammation and demyelination of peripheral motor and sensory nerve fibers. Early in its progression either upper or lower
motor signs may predominate. In almost all cases, patients with ALS show features of both UMN and LMN dysfunction.
Post-polio syndrome is a lower motor neuron (LMN) syndrome that does not present with sensory paresthesias and is
typically asymmetrical. MS will present with upper motor neuron signs: spasticity and hyperreflexia.

C40 C3 III
Slowing of pulse and increased venous pressure are possible.
Valsalva maneuver results from forcible exhalation with the glottis, nose, and mouth closed. It increases intrathoracic
pressures and causes slowing of the pulse, decreased return of blood to the heart, and increased venous pressure. A
cholinergic or vagal response is the result of parasympathetic nervous system (PNS) stimulation.

C41 C4 II
Arterial insufficiency
Arterial insufficiency can be determined by skin color changes during position changes of the foot, termed rubor of
dependency test. Chronic venous insufficiency can be determined by the history, presence of aching calf pain with
prolonged standing, a percussion test in standing, or Trendelenburg's test (retrograde filling test). With chronic
insufficiency, skin will be dark and cyanotic. Acute deep vein thrombophlebitis can be evident with aching calf pain,
edema, and muscle tenderness. Lymphedema is evident with visual inspection (i.e. swelling, decreased ROM) and
volumetric measurements.

C42 C8 II
Functional Reach of 7 inches.
All of these instruments can be used to examine functional balance and fall risk. A Functional Reach score of less than 10 is
indicative of increase fall risk. A POMA score of 27 out of a possible 28 is an excellent score (scores below 19 indicate a
high risk for falls while scores between 19-24 indicate moderate risk for falls). A Berg score of 50 out of a possible 56
points also indicates low fall risk. A Timed GUG score under 20 seconds for the 3 meter walk and turn test indicates low
fall risk (score of over 30 seconds indicate increased risk).

C43 C6 III
A Trendelenburg gait is characterized by a lateral trunk lean toward the side of the hip abductor weakness. It is a
compensatory strategy to keep the pelvis from dropping on the contralateral side during stance. The best intervention is
bridging which activate both hip extensors and abductors. The use of Theraband increases the loading of hip abductors.
Lateral leg slides (gravity eliminated position for hip abductors) would not sufficiently strengthen the hip abductors for
weightbearing function. The other choices do not encourage static holding of hip abductors.
C44 C3 II
Not use supplemental O2
A 70 year-old male would likely have a resting oxygen saturation of 95% from the changes associated with aging alone.
There is no need to supplement oxygen in this case. The guideline for supplemental oxygen is an SaO2 < 88%. Therefore,
the use of oxygen in this scenario is not justified. Supplemental O2 is by prescription only unless it is an emergency.

C45 C8 II
Lightweight wheelchair.
Ambulation at this level lesion requires too much energy and time to be functional. The parapodium permits standing but
does not allow for sufficient mobility for the entire school day. The lightweight wheelchair is the best choice for this child.

C46 C1 III
Because the left thoracic facet joint capsule is restricting movement, motion that would stretch the capsule would facilitate
improved right rotation. With right rotation the left superior facets move upward (opening the joint and stretching the
capsule) and right facets move downward (closing the joint and putting the capsule on relative slack). Additionally, trunk
flexion also moves the superior facets upward (opening the joint and stretching the capsule). Therefore, combined right
rotation and flexion will most effectively stretch the left facet joint capsule.

C47 C1 I
Soleus and gastrocnemius.
The soleus and gastrocnemius muscles oppose the dorsiflexion moment that exists at the ankle due to the line of gravity
which falls slightly anterior to the lateral malleolus.

C48 C7 III
What occurred, when and where it occurred and witness statements.
An incident report should avoid interpretive information such as cause of the occurrence or corrective actions that were
taken. The typical information included on an incident report is the name of those involved, inclusive of witnesses, what
occurred, when it occurred and where it occurred. There is no presumption that someone was injured.

C49 C5 III
US at 1 MHz
Pulsed ultrasound produces non-thermal effects (acoustic streaming, cavitation, microstreaming) which during the early
stages of inflammation process promotes tissue repair. The therapist would select 1 MHz for a deep therapeutic effect.

C50 C2 II
This patient demonstrates involvement of the long tracts (sensory and motor) indicative of involvement of the contralateral
cerebral cortex. The involvement of the face indicates a lesion above the level of the midbrain. A lesion in the spinal cord
would not affect the face. A lesion in the brainstem would produce facial signs contralateral to the limb signs.

C51 C2 I
Stocking and glove distribution of the lower and upper extremities.
Symmetrical involvement of sensory fibers, progressing from distal to proximal, is the hallmark of polyneuropathy. It is
termed stocking and glove distribution and is the result of the dying back of the longest fibers in all the nerves from distal
to proximal. Sensory symptoms include decreased sensation and pain, paresthesias and dysesthesias (abnormal sensations
such as numbness, tingling, or prickling). Allodynia refers to the perception of an ordinarily painless stimulus as painful
and is not characteristic of polyneuropathy.

C52 C3 II
A 2o AV heart block.
Criteria for reducing exercise intensity according to the American College of Sports Medicine include: 1) onset of angina
and other symptoms of exertional intolerance, 2) SBP equal to or greater than 240 mm Hg, DBP equal to or greater than
110 mm Hg, 3) greater than 1 mm ST segment depression, horizontal or downsloping, 4) increased frequency of ventricular
arrhythmias, 5) 2° or 3° AV block, or other significant ECG disturbances.

C53 C4 III
Begin with an interval walking program, exercising only to the point of pain.
This patient is exhibiting classic signs of chronic arterial insufficiency. Rehabilitation guidelines for arterial disease include
using an interval walking program of moderate intensity and duration, 2-3 times/day. The patient should be instructed to
exercise to the point of pain, not beyond. Exhaustive exercise and persistent pain are contraindicated. Calcium channel
blockers may be used in vasospastic disease; exercise is not contraindicated.
C54 C8 III
Clean and debride the wound, and apply a hydrogel dressing.
A stage II ulcer (deep ulcer) involves a partial thickness skin loss with involvement of epidermis, dermis or both; it is
reversible. Intervention should be directed toward improving perfusion and relieving localized pressure. The wound should
be cleaned with an antimicrobial agent, debrided of necrotic tissue, and covered with a sterile dressing. Hydrogel dressings
maintain moisture in the wound bed, soften necrotic tissue, and support autolytic debridement. Pressure relief is also an
important consideration. Techniques of protective foot care should be taught. A dry, sterile dressing is contraindicated as is
the application of skin lubricants.

C55 C6 III
A 17.5 inch seat height.
A hemi or low seat wheelchair has a seat height of 17.5 inches (20 inches is the standard seat height). The lower seat height
permits the patient to propel and steer the wheelchair using the sound upper and lower extremities. Elevating legrests may
be considered if the patient has problems with edema but should not be considered standard. Detachable arms are a useful
option but not the most appropriate in this example.

C56 C3 III
Give 2 rescue breaths followed by 30 chest compressions, repeating the cycle.
New guidelines form the American Heart Association (November 2005) concerning Basic Life Support specifies calling
911 for unresponsive adults before beginning CPR. The compression rate for adult CPR is about 100 per minute with a
recommended compression-to-ventilation ratio of 30:2. The old ratio was 15:2. Lay rescuers will be taught to no longer
check for pulse but to begin chest compressions immediately. If available, the healthcare rescuer should retrieve the AED
after calling for emergency services and if trained, use the AED after beginning CPR.

C57 C1 II
Descending stairs.
Loss of dorsiflexion will make descending stairs most difficult because the ankle must have dorsiflexion during the single
limb support phase during descent. Full range in dorsiflexion is not needed for the other choices.

C58 C6 III
Higher than the torque actually generated by the contracting hamstrings.
Gravity produced torque adds to the force generated by the hamstrings when it contracts, giving a higher torque output than
is actually produced by the muscle. Testing values may be misleading; software is available to correct for the effects of
gravity.

C59 C1 III
Central P/A pressure at a 60 degree angle on the spinous process of T6 while stabilizing T5.
In a closing restriction, the inferior facets of the superior vertebra will not inferiorly glide on the superior facets of the
inferior vertebra. Therefore T5 inferior facets will not caudally glide on the superior facets of T6. Stabilizing T5 and
application of pressure to T6 localizes the cephalad movement of the superior facets T6 on T5 bilaterally. The angle of the
thoracic facets is 60 degrees, therefore the application of force should be at the same plane.

C60 C2 II
Benign paroxysmal positional vertigo.
Benign paroxysmal positional vertigo (BPPV) is characterized by acute onset of vertigo and is positional, related to the
provoking stimulus of head movement. Vestibular neuritis is an inflammation of the vestibular nerve caused by a virus and
typically produces symptoms of dysequilibrium, nystagmus, nausea and severe vertigo. Meniere's disease is characterized
by a sensation of fullness in the ears associated with abnormal fluid build-up. Additional symptoms include tinnitus,
vertigo, nausea and hearing loss. Acoustic neuroma (vestibular schwannoma) produces unilateral sensorineural hearing loss
along with vestibular symptoms.

C61 C3 II
Increased heart rate and contractility at rest.
Atropine is an anticholinergic agent (it blocks the action of acetylcholine at parasympathetic sites in smooth muscle,
secretory glands, and the CNS). It produces an increase in heart rate and contractility and is used to treat symptomatic sinus
bradycardia, and exercise-induced bronchospasm. Major adverse cardiovascular reactions can include tachycardia,
orthostatic hypotension, palpitations, ventricular fibrillation, and increased ischemia in patients with myocardial infarction.
C62 C4 II
Hematuria and ecchymosis.
Warfarin sodium (Coumadin) is an anticoagulant, indicated in the prophylaxis and treatment of venous thrombosis,
pulmonary embolism, and thromboembolic disorders. Potential adverse reactions include hematuria and ecchymosis (skin
discoloration and hemorrhaging). Serious bleeding is possible with drug toxicity. Xeroderma (dry skin), cellulitis
(inflammation of tissues) and palpitations (awareness of heart rhythm abnormalities) are not seen as adverse reactions with
warfarin.

C63 C8 III
Highlighting steps with pastel colors of blues and greens.
Cataracts produce a clouding of the lens with gradual loss of vision. Patients experience difficulties in low light, high glare
environments, and in perceiving cool colors. Color coding with warm colors: reds, yellows or oranges is appropriate.

C64 C6 II
Standing, partial wall squats.
The quadriceps muscle is responsible for most of the energy generation needed to transfer up stairs to the next level. Partial
wall squats are the best choice to strengthen these muscles (closed chain exercise). During forward continuance
(corresponding to midstance), the ankle plantarflexors assist. Hip extensors are also active concentrically, assisting these
actions. The other choices might be good lead-up activities for gait but would not optimally strengthen the key muscles
involved in ascending stairs.

C65 C4 II
Postpone ambulation and report the findings immediately.
The patient is exhibiting early signs of acute deep vein thrombophlebitis (DVT). These findings should be reported
immediately. Exercise and ambulation are contraindicated during the acute phase. If DVT is present, the patient will be
given anticoagulation medication and will remain on bed rest with elevation of the involved leg until the acute phase
subsides.

C66 C8 I
Asking the child and his parents to describe the boy’s most serious functional limitations.
The child and his parents/caretakers play an important part in determining impairments, functional limitations, disability
and future interventions. Taking a thorough initial history is important in determining what the other components of the
examination should be. The incorrect choices, may indeed be appropriate; however, the interview process may help decide
which examination tools are needed.

C67 C1 I
Anteversion.
The pathology commonly associated with medial femoral torsion and toeing-in is hip anteversion due to an increase in the
angle (greater than 15 degrees) between the femoral condyles and neck of the femur.

C68 C1 III
Trunk extension and abdominal stabilization exercises.
Patients with osteoporosis must be educated in the area of prevention. They should avoid trunk flexion or rotation exercise
as it may facilitate a compression fracture of the spine. They also have a fear of falling and fractures, therefore it is very
important to strengthen from the core to the floor as well as train in proprioception and balance enhancement techniques.

C69 C1 II
Do a comprehensive examination, and if the therapist suspects abuse report the finds to the appropriate authorities.
According to APTA’s Guidelines for Recognizing and Providing Care for Victims of Domestic Violence this patient falls
into a category of high risk. Women between the ages of 17 and 28 years and woman who are single, separated, or
divorced or who are planning a separation or divorce are at high risk. Battered women usually have more than one injury.
Most injuries occur in the head, face, neck, breasts, and abdomen. According to AMA, battered women represent 23% of
pregnant women who seek prenatal care. The victim may not volunteer information about her situation, but more often
than not when asked they will reveal it. It is important for the physical therapist to identify resources available for victims
of domestic violence and to know their own state reporting laws. In most state jurisdictions a physical therapist may be
fined or indicted for failure to report.
C70 C5 III
High volt monophasic pulsed current.
Because high volt pulsed current is a monophasic, unidirectional current, the unidirectional current would produce a
therapeutic effect at the active (treatment) electrode. A negative charge (polarity) should be applied for a bactericidal effect
of a positive charge given to promote wound healing. A biphasic current, which alternates the polarity, would tend to
negate the treatment effects. Russian (burst) and interferential (beat) are medium frequency biphasic currents. Interrupted
currents (> 0.5 seconds interruption) are also not used for tissue healing.

C71 C5 III
Prone with no pillow.
Placing the patient in the prone position would better align the spine so that the pull of the traction would be along the axis
of the vertebral bodies. If this position is intolerable, then a pillow may be placed under the abdomen. Flexing the spine
could exacerbate the disc herniation. A supine, knee flexed position can be used for spinal stenosis.

C72 C2 III
Abduction with elbow extension
Hemiplegic synergies are present and should not be reinforced. Shoulder abduction with elbow flexion is part of the
flexion synergy while adduction with elbow extension is part of the extension synergy. Adduction with elbow flexion is an
out-of-synergy combination but does not strengthen the deltoid. Shoulder abduction with elbow extension is the correct
choice.

C73 C2 II
The sound side.
Unopposed action of the eye muscles causes the eye to deviate in the direction of the intact musculature. Thus, patients
with hemispheric lesions may look away from the hemiplegic side. Patients with brainstem lesions may look toward the
hemiplegic side.

C74 C3 II
ST segment depression from baseline of 3 mm horizontal or downsloping depression.
A positive exercise tolerance test (graded exercise test) indicates myocardial ischemia with increasing exercise intensities.
The optimal test duration is 8-12 minutes but can be terminated if symptoms of exertional intolerance are evident. The
American College of Sports Medicine indicates these include: 1) onset of moderate to severe angina, 2) a drop in SBP with
increasing workload, 3) serious arrhythmias, 4) signs of exertional intolerance (pallor, cyanosis, cold or clammy skin), 5)
unusual or severe shortness of breath, and 6) CNS signs (ataxia, vertigo, visual or gait problems, confusion). Relative
indications to stop the test include: 1) hypertensive response (equal to or greater than 260/115 and 2) ECG changes from
baseline (greater than 2 mm horizontal or downsloping; ST segment depression or greater than 2 mm ST segment
elevation). Source: ACSM’s Guidelines for Exercise Testing and Prescription, 6th ed, Lippincott Williams & Wilkins.

C75 C1 II
Closed chain terminal knee extension exercises.
The test that was conducted was a Lachman’s test to determine integrity of the anterior cruciate ligament. A positive test
suggests laxity of the anterior cruciate ligament. Quick cutting/lateral movements that occur in agility training and heavy
joint leading that occurs with plyometric exercise should be avoided until the muscular restraints that reduce excessive
anterior translation of the affected tibiofemoral joint are strengthened. Open-chain knee extension may place excessive
load on the ACL. Closed-chain terminal knee extension exercises are safe and effective secondary to the dynamic stability
inherent with this type of exercise.

C76 C4 III
Facilitate autolytic debridement and absorb exudate.
Moisture-retentive occlusive would dressings such as calcium alginate are recommended for use on exudating wounds
(grade III ulcer). They maintain a moist wound environment and do NOT allow rapid evaporation, absorb exudate, provide
autolytic debridement, reduce pain at the wound site, and promote faster healing (reepithelialization). A disadvantage is
that they are very permeable to bacteria, urine, and so forth. Semirigid dressings such as Unna’s boot provide little support.
C77 C4 III
Avoid direct exposure to blood and body fluids.
Hepatitis B is a viral infection that is transmitted by close contact with the infected patient’s body fluids (nasopharyngeal
exudate, saliva, sweat, urine, feces, semen, vagina secretions) and blood and blood products. Healthcare workers should be
vaccinated against the possibility of infection as they are in a high risk category and should carefully follow standard
precautions to protect themselves (refer to Chapter 5, Table 5-2). Droplet precautions are used when microorganisms can
be transmitted by the patient during coughing, sneezing or talking. Contact precautions are designed to reduce the risk of
microorganism transmission by direct or indirect contact.

C78 C6 II
Knee extensors.
The quadriceps are maximally active at heel strike (initial contact) to stabilize the knee and counteract the flexion moment.

C79 C3 II
Have the patient sit down, continue monitoring, and notify the physician immediately.
This tracing shows premature ventricular contractions (PVCs) that are multifocal (originating from different irritable
ventricular focus). These multiform PVCs pose a potential danger of deteriorating into ventricular tachycardia and
ventricular fibrillation (cardiac standstill). Since the heart is demonstrating a high degree of irritability, the best course of
action is to stop the exercise, have the patient sit down, continue monitoring carefully, and notify medical staff (attending
physician) immediately.

C80 C7 III
Practice stair climbing inside the parallel bars using a 3 inch step.
The most appropriate lead-up activity to promote the skill of stair climbing is practice using a 3 inch step in the parallel
bars. Passive movements do not promote active learning. Marching in place and isokinetic training may improve the
strength of the hip flexors but do not promote the same synergistic patterns of muscle activity as the desired skill.

C81 C3 III
Postural drainage, percussion, and shaking to the right lower lobe.
Viral pneumonia does not cause an intra-alveolar exudate; therefore, percussion and shaking are not indicated. All of the
other interventions could be beneficial to the patient.

C82 C4 II
Outline realistic short term goals to improve independence while maintaining for the possibility of further recovery.
This patient is exhibiting denial, the first stage in psychological acceptance (Kubler-Ross). Denial can be protective,
particularly in the early phases. However, this patient’s denial persists three months after injury and is limiting her rehab
progress and outcomes. It is important to provide a message of hope tempered with realism. Encouraging short term goals
to improve function is the most appropriate choice.

C83 C8 III
Reposition the child in a proper sitting position using postural supports.
Feeding can only be successful if the child is positioned in a stable sitting posture: head upright, trunk erect with pelvis
neutral and hips flexed to 90 degrees, and feet resting flat. Correct positioning in sitting will facilitate upper extremity
function (grasp and release) as well as swallowing.

C84 C1 III
Inferior glide at 55 degrees of abduction.
The convex-concave rule for mobilization applies. The most effective position to mobilize for improved shoulder
abduction is in the resting position (55 degrees). Because the convex humeral head is moving on the concave glenoid, an
anterior glide would be most appropriate to improve shoulder abduction.
C85 C1 II
Capsule-ligamentous pattern of TMJ on the left.
The capsule-ligamentous pattern of the TMJ is limitation on opening, lateral deviation greater to the uninvolved side, and
deviation on opening is to the involved side. Weakness of the lateral pterygoids presents as deviation on protrusion to the
opposite side of the muscle weakness. A capsular pattern of the cervical spine presents as side flexion and rotation equally
limited and extension. Normal parameters for TMJ measures are 25-3C8 III
Reposition the child in a proper sitting position using postural supports.
Feeding can only be successful if the child is positioned in a stable sitting posture: head upright, trunk erect with pelvis
neutral and hips flexed to 90 degrees, and feet resting flat. Correct positioning in sitting will facilitate upper extremity
function (grasp and release) as well as swallowing.

C84 C1 III
Inferior glide at 55 degrees of abduction.
The convex-concave rule for mobilization applies. The most effective position to mobilize for improved shoulder
abduction is in the resting position (55 degrees). Because the convex humeral head is moving on the concave glenoid, an
anterior glide would be most appropriate to improve shoulder abduction.

C85 C1 II
Capsule-ligamentous pattern of TMJ on the left.
The capsule-ligamentous pattern of the TMJ is limitation on opening, lateral deviation greater to the uninvolved side, and
deviation on opening is to the involved side. Weakness of the lateral pterygoids presents as deviation on protrusion to the
opposite side of the muscle weakness. A capsular pattern of the cervical spine presents as side flexion and rotation equally
limited and extension. Normal parameters for TMJ measures are 25-35 mm functional, and 35-50 mm normal, normal
protrusion is 3-6 mm, and normal lateral deviation is 10-15 mm.

C86 C7 III
Deny access to the chart unless written permission by his wife is granted.
The issue here is patient confidentiality. Spouses do not have access to medical information unless they have consent of
proxy because of incompetence of the other spouse. In accordance with APTA’s Guide for Professional Conduct Principle
1. Physical therapists respect the rights and dignity of all individuals. Specifically 1.1 Attitudes of Physical Therapists B.
Physical therapists are to be guided at all times by concern for the physical, psychological and socioeconomic welfare of
those individuals entrusted to their care; and 1.2 Confidential Information C. Information derived from the working
relationship of physical therapists shall be held confidential by all parties. HIPAA regulations also limit access to medical
record information unless consent is granted.

C87 C3 II
Pneumothorax
While all of the pathologies listed would cause panic on the part of the patient, mucous plugging of an airway would not
cause pain. The deviation of the trachea would not result from angina or pulmonary emboli, but would happen with a
pneumothorax and lung tissue collapse (which could result from mucous plugging). The deviation of the trachea toward
the right, with the chest pain on the left, is a match of symptoms for the occurrence of a pneumothorax on the left. The
history of a rib fracture makes pneumothorax all the more likely.

C88 C5 III
Slight abduction and internal rotation.
Abduction and internal rotation of the shoulder places the supraspinatus tendon in a good position to apply ultrasound by
exposing the tendon from under the acromion process.

C89 C2 III
Reach forward to bear weight with the right arm extended against the wall.
Stage 4 is characterized as some movement combination that do not follow the paths of either synergy. The influence of
spasticity is beginning to decline. Weightbearing on extended arm in sitting is an early training activity (stage 2-3) to
promote shoulder stability while keeping the arm out of its typical spastic pattern. Reaching overhead with the elbow
extended is an advanced activity, characteristic of stage 5 recovery. Putting on socks and shoes is also an advanced activity
for the upper extremity requiring manual dexterity. Reaching forward and weightbearing with the shoulder at 900 and
elbow extended is the best choice for stage 4 training.
C90 C2 II
Broca’s aphasia.
This patient is demonstrating classic signs of Broca’s aphasia (also known as nonfluent, expressive, or motor aphasia). It is
the result of a lesion involving the third frontal convolution of the left hemisphere. Broca’s aphasia is characterized by
slow and hesitant speech with limited vocabulary and labored articulation. There is relative preservation of auditory
comprehension. Wernicke’s aphasia is characterized by impaired auditory comprehension and fluent speech. Global
aphasia is a severe aphasia with marked dysfunction across all language modalities. Dysarthria is an impairment in the
motor production of speech.

C91 C2 III
Engage in a calming activity and document the behaviors.
Patients in Level IV of recovery are confused and agitated. Behavior is bizarre and nonpurposeful relative to the immediate
environment. This patient is unable to cooperate directly with formal examination or treatment, lacking both selective
attention and memory. The therapist needs to observe and document the behaviors closely and engage the patient in a
calming activity such as slow rocking. A quiet, closed environment is critical.

C92 C3 I
Left ventricular failure.
Clinical manifestations of left ventricular failure (congestive heart failure), include: those described in the case example
along with an S3 heart gallop, paroxysmal nocturnal dyspnea, orthopnea, and signs and symptoms of pulmonary edema
(marked dyspnea, pallor, cyanosis, diaphoresis, tachypnea, anxiety, and agitation). Typical clinical manifestations of right
ventricular failure include: dependent edema of the ankles (usually pitting edema), weight gain, fatigue, right upper
quadrant pain, anorexia, nausea, bloating, right sided S3 or S4, cyanosis of nail beds and decreased urine output.
Impending MI may include anginal pain or discomfort in the chest, neck, jaw or arms, palpitations, tachycardia, or unusual
fatigue or dyspnea. Pericarditis produces substernal pain that may radiate to neck and upper back, difficulty swallowing,
pain aggravated by movement or coughing and relieved by leaning forward or sitting upright, and a history of fever, chills,
weakness, or heart disease.

C93 C3 II
Prolonged endurance training has resulting in a low heart rate.
One of the benefits of endurance training is decreased heart rate and blood pressure with improved functional capacity. This
patient demonstrates a significant training effect, most likely from months or years of consistent running. Low heart rate is
not a usual finding with most cardiovascular pathology.

C94 C4 II
Atrophy and osteoporosis.
Prolonged use of corticosteroids may result in muscle weakness, osteoporosis, fractures and joint pain. Large doses are
associated with Cushingoid changes (moonface, central obesity, hypertension, myopathy, electrolyte and fluid imbalance,
and so forth). Common CNS changes include insomnia and nervousness.

C95 C4 I
The infected wound can convert the area to a full thickness burn.
A deep partial thickness burn will heal in about 3-5 weeks if it does not become infected. An infection typically results in
conversion of the wound to a full thickness burn. Full thickness burns are without sensation as the nerve endings are
destroyed. However, the area is not pain free as adjacent areas of partial thickness burns have intact nerve endings and can
experience pain. The risk of hypertrophic and keloid scars is high with deep partial thickness or full thickness burns. With
wound conversion, grafting will be necessary as all epithelial cells are destroyed with a full thickness burn.

C96 C6 III
Viscoelastic shoe insert with forefoot lateral wedge.
Supination of the foot (pes cavus) is accompanied by supination of the talocalcaneonavicular (TCN), subtalar and trans-
versal tarsal joints. It is characterized by an abnormally high arch. The flexible cavus foot generally responds well to
orthotic foot control especially in a young child. The best choice is a viscoelastic shoe insert with forefoot lateral wedge.
The other choices are used to control flexible pes valgus.

C97 C6 III
Stand pivot transfer to the sound side.
During initial healing, it is important to protect the hip from dislocation or subluxation of the prosthesis. With a
posterolateral incision. excessive hip flexion and adduction past neutral are contraindicated. This is minimized by
transferring to the sound side. Full ROM of the operated hip is also contraindicated.
C98 C6 III
A rigid frame.
Sports competition wheelchairs are usually made with rigid construction and very strong lightweight materials. A folding
wheelchair does not provide the stability needed for competition sports. Pneumatic (air-filled) tires provide a smoother ride
and improved traction as opposed to hard-rubber tires. A low seat back enhances the user's upper body/arm movements. A
higher seat back is indicated for patients with decreased trunk control (not a factor in this example).

C99 C8 III
Sideward protective extension in sitting.
Sideward protective extension in sitting is a functional, protective reaction which normally occurs at about the same time as
sitting begins. The child who is starting to sit should already have prone and supine tilting reactions. It is too early to begin
standing tilting reactions.

C100 C3 II
40 to 85% HR reserve (Karvonen’s formula).
This patient has impaired functional abilities as noted by the low number of feet traveled in 12 minutes. He also has a high
resting heart rate, likely the result of his pulmonary medications. To correctly prescribe exercise for this patient, his high
resting heart rate needs to be taken into consideration. Karvonen's formula, the heart rate reserve method, uses his high
resting heart rate as part of the formula. [(HRmax - HRrest) x 40 to 85%] + HRrest. = Target heart rate range. If the therapist
were to use the formula 70 to 80% of HRmax, the therapist would find that part of his target heart rate for exercise was lower
than his resting heart rate. Using a predicted number for heart rate when an actual exercising heart rate max is known is an
inaccurate way of prescribing exercise. Finally, since 1) the MET charts are based on a healthy population, 2) the therapist
has the actual heart rate data from the test and, 3) the therapist doesn't have actual metabolic equivalents during the exercise
test, there is no reason to use METs in the calculation of exercise intensity.

C101 C8 II
Independent in wheelchair mobility.
Duchenne's muscular dystrophy is a progressively debilitating disease with the majority of teenagers becoming wheelchair
bound but independent. A motorized tilt-in space wheelchair is a good choice.

C102 C1 I
9-11
The most effective age to screen girls for scoliosis is just before the pubescent growth spurt between 9-11 years when the
scoliotic curve can increase dramatically. Boys should be screened between 11 to 13 years of age due to differences in the
age of onset of puberty between girls and boys.

C103 C1 III
Lower cervical flexion.
Flexion increases the space at the intervertebral foramen, allowing the C6 nerve root to decompress and reduce or alleviate
radicular pain.

C104 C1 I
Tibial, femoral, and pelvic internal rotation.
With the patient standing, the calcaneus is fixed to the ground. Subtalar joint pronation will occur as the talus plantar flexes,
adducts and inverts. In response to subtalar joint pronation, obligatory internal rotation of the tibia, femur, and pelvis
occurs.

C105 C7 III
According to APTA's Guidelines for Physical Therapy Documentation: Charting errors should be corrected by drawing a
single line through the error and initialing and dating the chart. The appropriate mechanism for electronic documentation
clearly indicates that a change was made without deletion of the original record.

C106 C5 III
Pulse duration.
Decreasing the pulse duration reduces the electrical charge of each pulse making the current more comfortable by
decreasing the total current applied while maintaining the full therapeutic effect.
C107 C2 II
Horner’s syndrome
Horner's syndrome occurs with lesions of the lateral medulla or cervical sympathetic chain. Symptoms include miosis,
ptosis, and anhidrosis over the face. Argyll Robertson pupil is a small, fixed size pupil that does not react to light but does
contract with accommodation. It is caused by neurosyphilitic lesions interrupting fibers from parasympathetic nuclei
(Edinger-Westphal nuclei) of the oculomotor nerve. Homonymous hemianopsia is a field detect involving loss of visual
information from one hemifield (common in cortical lesions and stroke). Nystagmus is abnormal oscillating eye movement.

C108 C2 II
Vestibulocerebellum
The symptoms are suggestive of cerebellar dysfunction. The vestibulocerebellurn (archicerebellum) is concerned with
adjustment of muscle tone in response to vestibular stimuli. It coordinates muscle actions to maintain equilibrium. The
spinocerebellum (paleocerebellum) controls muscle tone and synergy of muscles on the same side of the body.

C109 C3 I
Peripheral arterial disease.
Intermittent claudication, often the earliest indication of peripheral arterial disease (PAD), is manifested by cramping, pain,
or fatigue in the muscles during exercise, which is typically relieved by rest. The calf muscle is most commonly affected,
but discomfort may also occur in the thigh, hip, or buttock. Cessation of pain immediately upon stopping the exercise is
characteristic of intermittent claudication, not other spinal problems. With severe disease however, pain may be present
even at rest. The findings cannot explain the incorrect choices.

C110 C4 II
Discontinue treatment and notify his physician immediately.
This patient is demonstrating signs and symptoms of aortic aneurysm. Pain is intermittent or constant and can be lei! in the
midabdorninal or low back regions. The pulsating mass is highly significant and his level of hypertension dramatically
increases his risk of rupture. This is a serious medical condition; you should notify his physician immediately. You should
not rely on the patient to contact his physician. PT intervention should cease.

C111 C4 III
Whirlpool with povidone-iodine.
Wound irrigation or lavage is an effective way to remove debris and contaminants, and reduce bacterial counts on wound
surfaces. Whirlpool can be used as a means of wound cleansing and mechanical debridement. Antibacterial agents are
typically used, e.g., povidone-iodine (Betadine) or Chloramine-T (Chlorazine) if the wound is purulent. Limitations of
whirlpool include risk of contamination and use of the dependent position can increase venous congestion and edema.
Water temperature should be warm (not 20 degrees Celsius) to stimulate peripheral circulation. Hydrogen peroxide is
indicated to debride wounds with large amounts of necrotic tissue and is not used for wound lavage. Dakin’s solution is an
appropriate bactericidal agent; however, it is not administered while the patient is in the whirlpool.

C112 C8 I
Sundowning behavior
A patient with Stage 2 Alzheimer’s disease, can be expected to exhibit impaired cognition and abstract thinking,
sundowning (defined as extreme restlessness, agitation, and wandering that typically occurs later afternoon), inability to
carry out ADL, impaired judgment, inappropriate social behavior, lack of insight, repetitive behavior and a voracious
appetite. Inability to communicate is characteristic of Stage 3. Short term memory loss and disorientation to time and date
are early signs of the disease (Stage 1).

C113 C6 III
Provide posterior directed resistance to the right ASIS during stance.
Stride length is the linear distance between heel strike of one lower extremity to the next heel strike of the same extremity.
Resistance applied in a posterior direction to the right ASIS during stance will facilitate forward pelvic rotation and
enhance stride length. Anterior directed resistance functions to pull the hip forward but does little to facilitate active
motions. The gluteals function to stabilize the limb.

C114 C6 III
Prone-on-elbow pushups.
Prone-on-elbow pushups strengthen glenohumeral and scapular musculature required for pushups in sitting. While
strengthening of the triceps (intact in this patient) is also important, achieving proximal stability first is critical to promoting
control. Supine-on-elbows activities are used to assist in bed mobility and to prepare the patient to assume the long sitting
position.
C115 C7 III
Level 1.0 Receiving
In the affective domain, level 1.0 objectives deal with attending to phenomena and stimuli. This student lacks these
foundation behaviors in the affective domain and is thus functioning at a very low level. His problem is not one of
behaviors at the higher levels of the affective domain (all other choices).

C116 C7 III
Have the same therapist reassess the patients after 6 months.
Reliability is the degree to which a test consistently measures what it is intended to measure. Intrarater reliability is
established by having the same rater measure on multiple measurement trials. Intrarater reliability can be established if all
raters are trained to administer the measurement instruments and demonstrate consistency of rating (there is no evidence of
this).

C117 C3 II
Heart rate.
An exercise tolerance test should be performed prior to commencing an exercise program for all high-risk individuals. The
best measurement of exercise intensity in a newly exercising individual is heart rate. Rating of perceived exertion (RPE)
will become a valuable measurement tool once the patient becomes adept at using it, but it would not be reliable for the first
exercise session. MET level is more of a measurement of workload, not an accurate measurement of an individual’s
response to exercise. Respiratory rate is not used to prescribe exercise intensity.

C118 C8 II
Explain that this is normal and that the stepping was a newborn reflex that has gone away.
The mother probably saw the neonatal stepping reflex which is normal in a newborn but is not exhibited in the older infant
probably because of anthropomorphic factors and neural maturation. The age this reflex typically disappears is 2 to 3
months of age. In most children pull-to-stand emerges at 8-9 months while unassisted standing and walking occurs 10-15
months. Stepping at this age is not the result of a reflex.

C119 C1 I
35-44 mm.
Average AROM is approximately 35 to 50mm. However, only 25-35mm of opening between the teeth is required for
normal everyday activity. The TMJ's are considered hypomobile if less than 25mm of opening is achieved. Hypermobility
would include values >50 mm.

C120 C1 III
Medial forefoot and rearfoot varus posting
Forefoot varus is a deformity such that the forefoot is in a position of inversion in relation to the rearfoot when the subtalar
joint is in the neutral position. Forefoot varus results in an abnormal gait pattern when abnormal compensatory subtalar
joint pronation appears, allowing the medial metatarsal heads to contact the ground. Excessive calcaneal eversion, talar
adduction, and talar plantar flexion occur. Correction of forefoot varus with excessive subtalar pronation can be achieved
with a customized orthosis. A rearfoot varus post corrects the excessively everted calcaneus closer to neutral. The medial
forefoot post (medial wedge) allows weight bearing on metatarsal heads.

C121 C1 II
Pronator teres syndrome
All of the above special tests are used to determine neurological compromise of the lower trunk and brachial plexus.
Special tests to rule out pronator teres syndrome are: 1) passive supination to elongate the pronator which is tight; this
would compress the nerve at that level, and 2) active resistance of pronation which would compress the nerve as it courses
through the pronator muscle belly.
C122 C7 III
Treat the patient, and bill for the 20 minute session given.
The physical therapist must deal with the principles of beneficence (the duty to foster the patient's interests), non-
maleficence (the duty not to harm the patient) and justice (the duty to distribute services equitably). The therapist is also
bound by Principles 4 and 7 of the APTA's Code of Ethics. Principle 3: Physical therapists accept responsibility for the
exercise of sound professional judgment and Principle 5: Physical therapists seek remuneration for their services that is
deserved and reasonable. The Guide for Professional Conduct farther interprets this principle in Principle 7.1 .a Business
and Employment Practices. Physical therapist's professional practices and their adherence to ethical principles shall take
precedence over business practices. Provisions of services for personal financial gain rather than for the need of the
individual receiving their services are unethical. Fees for physical therapy services should be. reasonable for the service
performed, considering the setting in which it is provided, practice costs in the geographic area, judgment of other
organizations, and other relevant factors.

C123 C5 III
Large electrodes, widely spaced.
Two reasons to place a largo electrode on a largo muscle are to: 1) stimulate a large number of muscle fibers and 2) reduce
the current density generated under each electrode making the current more comfortable to the patient. A wide
interelectrode spacing will allow the current to stimulate deep muscle fibers.

C124 C2 III
Supine, trunk in midline with small pillow under the scapula, arm extended on supporting pillow, and a small towel roll
under the knee.
In the upper extremity, spasticity is typically strong in scapular retractors, shoulder adductors, depressors and internal
rotators; elbow flexors and forearm pronators; wrist and finger flexors. In the lower extremity, spasticity is typically strong
in hip and knee extensors, pelvic retractors, hip adductors and internal rotators; hip and knee extensors; plantar flexors. In
the trunk, lateral flexors are typically spastic. Supine with a small pillow under the scapula, arm extended and small roll
under the knee represents the best choice to control for the expected spasticity.

C125 C2 I
Somatosensory integrity.
The term somatosensation refers to conscious relay pathways for discriminative touch, conscious proprioception, fast pain
and discriminative temperature. Sensory examination must rule out vision in order to establish the reliability of sensory
testing. Vestibular/visual/somatosensory integration can only be established by a series of tests that include both eyes open
and eyes closed, flat and compliant or moving surfaces (see Clinical Test for Sensory Interaction in Balance, Shumway-
Cook and Horak).

C126 C3 II
Congestive heart failure.
An S3 heart sound (ventricular gallop) is indicative of diastolic overload of the ventricles and decreased ventricular
compliance. It is the hallmark sign of congestive heart failure (left-sided failure) and can be heard during early diastole after
S2 Murmurs (aortic ejection sound, and pulmonic ejection sounds) are indicative of aortic and pulmonary valve dysfunction.
A pericardial friction rub is heard with pericarditis.

C127 C3 II
HR reserve formula and Ratings of Perceived Exertion.
Expected changes in the elderly include a lower VO2 max which is evidenced by a lower maximal HR, decreased muscle
strength with increased fatigability, and decreased neuromuscular coordination and balance. High variability exists for
maximal heart rates in persons over 65 years of age; the HR reserve method (Karvonen formula) and RPE is recommended.
MET levels vary considerably from individual to individual and cannot be relied on; 8 METs represents slow jogging and is
not appropriate for this individual.

C128 C4 II
Discontinue UBE exercise; use massage and active assistive ROM.
This patient presents with the classic signs and symptoms of early stage Complex Regional Pain Syndrome, Type I (reflex
sympathetic dystrophy). Edema can be successfully managed with a combination of elevation, massage and compression
bandaging. Active assistive ROM is important to maintain range. Immobilization through splinting is contraindicated. Ice
may be helpful for pain relief; however, it will not reduce the pitting edema and is not the best course of action.
C129 C8 II
Poor light adaptation.
Poor light adaptation (e.g. going from well lit outside stairs to a dark inside environment) is a characteristic finding in the
elderly. Presbyopia (loss of accommodation and near vision) results in reading difficulties. Safely training should include a
suggestion to leave a night light on.

C130 C6 III
Sliding board.
This patient does not have the required strength to stand against gravity. A sliding board transfer allows the patient to
perform the transfer independently. A dependent 1-man squat transfer (football transfer) is performed with the patient’s
arms around the therapist's waist and head down with legs flexed and off the ground. The therapist does all the work in
lifting and pivoting the patient. This is not appropriate for this patient. Stand-by assist is equally inappropriate.

C131 C6 III
Lift the front caster and ascend in a wheelie position
Curbs are ascended in the wheelie position (front casters lifted and moving first). Using momentum from several pushes
helps to assist elevation. Patients can descend backwards but must use a tucked forward lean position in order to prevent
falling backward or can descend forward using a wheelie position.

C132 C3 II
Decreased bone density.
While many of the choices are sequelae to long term systemic steroid use, the only one that is a contraindication to
percussion would be decreased bone density, as a rib fracture might be more possible. An increased blood pressure,
probably higher than that reported in the scenario, might be a contraindication to postural drainage. It is not a
contraindication to percussion. Barrel chest is seen in patients with emphysema not asthma.

C133C1 II
A Pancoast tumor.
A Pancoast tumor is an apical tumor that is typically found in conjunction with a smoking history. The clinical signs and
symptoms can be confused with neurovascular compromise at the level of thoracic outlet. The patient's smoking history,
rapid onset of clinical signs and symptoms, and pleuritic pain is highly suspect of an apical tumor. The pain reported is not
typical of Guillain-Barré syndrome. Thoracic outlet syndrome does not typically present bilaterally. Foraminal stenosis in
the midcervical spine will not cause weakness in the hand intrinsics. The smoking history and hoarse voice coupled with
the other symptoms make this a “red flag” situation and suspicions of a tumor must be reported immediately.

C134
Atraumatic shoulder instability.
The patient's signs and symptoms describe an atraumatic instability of the shoulder. A patient with a rotator cuff lesion
would present with pain and weakness after eccentric load; protective shoulder hike may be present; weakness of abduction
or rotation or both with AROM; during PROM, pain would be elicited if impingement occurs; pain and weakness on
abduction and lateral rotation; resisted isometric movements with a positive drop arm, and empty can test. An impingement
problem would present with positive Neer and Hawkins-Kennedy impingement tests. The traumatic anteriorly dislocated
shoulder patient would present with apprehension and decreased ROM in abduction and lateral rotation with AROM;
muscle guarding and decreased ROM in apprehension position with PROM; resisted isometric tests produce pain in
abduction and lateral rotation, with the following positive special tests-apprehension, augmentation, and relocation.

C135 C5 III
Readjusting the harness and continuing with the treatment.
Readjusting the cervical harness angle to increase the force at the back of the skull (occipital protuberance) and lessen the
force on the mandible would decrease the pressure on the TMJ and make the treatment more comfortable for the patient.

C136 C6 II
Reposition the height of the shelf and items to below shoulder height.
Workstations should be designed to accommodate the persons who actually work on the job. Workstations should be easily
adjustable and designed to be comfortable for the worker.
C137 C2 III
D2 flexion.
The supraspinatus, infraspinatus, teres minor, and subscapularis comprise the rotator cuff and function to provide dynamic
stability to the shoulder. The supraspinatus assists in abduction while the others act during abduction to compress the
humeral head into the glenoid fossa. D2 flexion is the optimal pattern because all the rotator cuff muscles are activated. In
Dl flexion the infraspinatus and teres minor are activated but not the supraspinatus. Both Dl and D2 extension emphasize
internal rotation.

C138 C6 II
Cushion heel with a rearfoot valgus post.
This patient is presenting with symptoms of plantar fasciitis and heel pain. A cushion heel with medial forefoot and rear-
foot posts that promote a neutral foot position is the best choice for intervention. A UCBL insert is prescribed to realign a
flexible flat foot. A metatarsal pad is used to transfer stress from painful metatarsal heads.

C139 C3 II
2+ pulses.
A bruit is a swishing sound that occurs in the presence of narrowing of an artery. It is a characteristic finding of PVD.
Pulses may be decreased (l+/diminished or 0/absent); 2+ is normal.

C140 C4 II
Hepatitis B.
Healthcare workers are most likely to contract hepatitis B (estimated incidence 300,000 new acute cases in U.S. each year).
Transmission is through exposure to blood and blood products and infected body fluids. Hepatitis A has a much lower
reported incidence (35,000 new cases each year) and is transmitted primarily through the fecal-oral route and contaminated
food or water. HIV and tuberculosis also have lower incidences. HIV has a similar route infection as hepatitis B while
tuberculosis is an airborne infectious disease. See Table 5.1 Standard Precautions.

C141 C4 II
Rheumatoid arthritis.
Rheumatoid arthritis (RA) is characterized by morning stiffness, pain, and relatively symmetric joint involvement.
Laboratory abnormalities in RA include: positive serum rheumatoid factor (RF) and elevated erythrocyte sedimentation rate
(ESR). Articular and extrarticular manifestations include weight loss, malaise, nodulosis and vasculitis. Synovial fluid
analysis reveals elevated white blood cell count and protein count. Osteoarthritis (OA) or degenerative joint pain, and
fibromyalgia both produce pain but do not produce the laboratory findings reported above or nodulosis. Systemic lupus
erythematosus (SLE) is an immunologic disorder characterized by inflammatory lesions in multiple organ systems. It is
diagnosed by client history (multiple organ involvement, especially of skim, joints, serous membranes), systemic symptoms
(fever, malaise, fatigability, etc) and appearance of skin rash (erythema). ESR is elevated in patients with SLE, however,
nodulosis and joint malformations are not expected.

C142 C6 I
Prosthetic knee set too far anterior to the TKA line.
In order to increase stability of the knee, the prosthetic knee is normally aligned 'posterior to a line extending from the
trochanter to the ankle (TKA line). A knee set anterior to the TKA line will buckle easily. The gluteus medius contributes
to stability during stance, primarily lateral stability. An extension aid assists knee extension during the latter part of swing
phase.

C143 C6 III
Contoured foam seat.
A contoured foam seat that accommodates to the patient's body contours and provides an adequate seat base and lateral
support to stabilize the scoliosis provides the best option. A sling seat tends to increase asymmetries. Increasing the seat
depth encourages a posterior pelvic tilt and increases kyphosis. A firm seat with lateral knee positioners does not address
her trunk problems.

C144 C6 III
Onto her walker and one leg, tuck her pelvis by extending the upper trunk, and swing her other leg through.
The correct sequence for using a reciprocating gait orthosis (RGO) is to 1) shift weight onto one leg, 2) tuck the pelvis by
extending the upper thorax, 3) press on the crutches or walker; and 4) allow the other leg to swing through.
C145 C7 III
Involve him in goal setting and have him participate in structuring the training session.
An andragogical approach is best. He is an adult learner who should be allowed to share in the responsibility for planning
his learning experience. The therapist should help clarify the problem, structure the learning environment, and provide
necessary resources. Passing the buck or transferring responsibility to another is not appropriate.

C146 C7 III
The activities of the non-rehab group were not properly monitored and may account for these results.
To ensure adequate control, the researcher should attempt to remove the influence of any variable other than the
independent variable in order to evaluate its effect on the dependent variable. In this study, the investigator did not
adequately investigate the usual activities of the control group. The small number of subjects may also have contributed to
lack of significance.

C147 C6 II
Lateral lean toward the right.
Lateral trunk bending toward the orthotic stance leg can result from 1) excessive height of the medial upright, 2) weak
gluteus medius (Trendelenburg gait), 3) abduction contracture, or 4) a short leg. Posterior trunk bending is associated with a
weak gluteus maximus while anterior trunk bending is associated with a weak quadriceps.

C148 C6 II
A solid ankle AFO
A solid ankle AFO is indicated to control foot drop in the presence of spasticity. A grade of 2 (Modified Ashworth Scale)
indicates marked increase in muscle tone throughout the range, but the affected part can be moved. A dorsiflexion assist
and spiral AFO are contraindicated as they fail to adequately control spasticity and can actually increase spasticity. An
anterior or dorsiflexion stop limits dorsiflexion and can be used with weakness of the plantarflexors (not a problem in this
case).

C149 C8 II
Delay in achieving developmental milestones.
The 12-15 month-old child should be ambulating. At 24 months lack of ambulation is indicative of developmental delay.
Other developmental milestones include: sitting 6 mo. and creeping, pull-to-stand and cruise at 8-9 months.

C150 C1 III
Transverse processes of T8.
The axis of motion for the mid-thoracic vertebrae is above the spinous processes and below the transverse processes.
Therefore, if down-gliding/closure of T7-8 vertebral segment is required, the therapist's hand placement should be at the
transverse process of T8 or the spinous process of T7.

C151 C1 II
Thoracic outlet syndrome
Thoracic outlet syndrome (TOS) is a neurovascular compression syndrome affecting the upper quadrant. Due to the lack of
a good definitive objective test to confirm the presence of TOS, the diagnosis is a clinical one largely reached by exclusion.
Ulnar nerve entrapment typically presents with nocturnal numbness, never having sensory loss in the proximal or middle
portions of the forearm and no atrophy of intrinsic muscles innervated by the median nerve in the thenar eminence. Carpal
tunnel syndrome presents with thenar atrophy and sensory loss in the first two digits differentiated from TOS by
electrophysiologic studies indicating distal and not proximal compression. Pronator teres syndrome symptoms arise
primarily at the elbow with radiation into the radial aspect of the hand and numbness that extends into the median nerve
distribution. With passage of time atrophy of the thenar muscles will occur.

C152 C5 II
A vertebral artery test.
The vertebral artery test checks the integrity of the blood flow through the artery in the cervical region. Since the patient is
experiencing symptoms of circulatory disturbance and a unilateral pull could compress the left cervical structures, the
vertebral artery test is an appropriate screening test. The test consists of passively placing the patient's head in extension
and side flexion. Then the head and neck is slowly rotated to the laterally flexed side and held for 30 seconds. Some of the
positive signs may be syncope, lightheadedness, nystagmus or visual disturbances. Lhermitte’s sign is pain down the spine
and into the upper or lower limbs with passive flexion of the neck. It is used to identify dysfunction of the spinal cord
associated with upper motor neuron lesions and is typically positive in MS. Adson's maneuver is a test for thoracic outlet
syndrome. The Oppenheim test involves running a fingernail along the crest of the tibia: a positive lest is the same as a
positive Babinski
C153 C2 III
Ambulation using bilateral AFOs and canes
A spinal cord lesion at the level of L2 is considered a lower motor neuron injury (cauda equina injury). Intact movements
include hip flexion, hip adduction, and knee extension. This patient can be expected to be a functional ambulator using
bilateral ankle-foot orthoses and crutches or canes. For some community activities, the patient may elect to use wheelchair
for convenience and energy conservation but is not expected to be a full-time wheelchair user. Orthotic bracing of the
knees is not needed.

C154 C2 I
A series of letters traced on the hand.
Graphesthesia is the ability to recognize numbers, letters, or symbols traced on the skin. Barognosis is the ability to rec-
ognize different weights placed in the hand using identically shaped objects. Pallesthesia is the ability to recognize vibra-
tory stimuli, i.e. a vibrating tuning fork placed on a bony prominence. Stereognosis is the ability to recognize different
objects placed in the hand and manipulated. During testing, vision is occluded.

C155 C2 III
Light resistance to forward pelvic rotation during swing.
An elevated and retracted pelvis is a common problem during gait for many patients recovering from stroke. Providing light
resistance to forward pelvic rotation actively engages those muscles and reciprocally inhibits the spastic retractors.
Providing anterior directed pressure during swing or resistance to pelvic elevation may only serve to increase abnormal
tone in those muscles. The problem is a swing phase deficit; downward compression during stance is inappropriate for this
problem.

C156 C2 II
XII
The hypoglossal (C.N. XII) controls the movements of the tongue. Ipsilateral wasting and the deviation to the ipsilateral
side on protrusion are indicative of damage. Involvement of the glossopharyngeal (C.N. IX) results in slight dysphagia, loss
of taste in the posterior third of the tongue and loss of gag reflex. Involvement of the accessory (C.N. XI) results in minor
problems in deglutition and phonation along with weakness in ipsilateral shoulder shrugging. Involvement of the vagus
(C.N. X) results in dysphagia, hoarseness and paralysis of the soft palate.

C157 C3 II
Inferior wall.
ST elevation and T wave inversion on the ECG are the hallmarks of myocardial infarction. When they occur in leads II, III,
and AVF, it is most likely an inferior infarct. Lateral wall infarcts produce similar changes in I, AVL, V5 and V6; anterior
wall infarcts produce changes in V1, V2, V3 and V4; posterior wall infarcts produce changes in V1 and V2 with tall R and
T waves and ST depression.

C158 C4 I
All values are abnormal.
The hematocrit value is abnormally low; women average 42% with a normal range from 37% to 47%. The hemoglobin
value is also abnormally low; women average 12-to 16-g/100 ml of blood. The low values are most likely doc to blood loss.
The serum WBC (leukocytes) is abnormally high; normal values are 5,000 10 10,000/mm³. The elevated count (greater than
10,000) indicates acute infection.

C159 C4 II
Dermatitis
This patient is exhibiting symptoms of contact dermatitis. Primary treatment is removal of the offending agent (in this case
the total contact prosthesis) and treatment of the involved skin with lubricants, topical anesthetics, and/or steroids. He may
require a thin sock if the problem does not resolve. Cellulitis is a suppurative inflammation of the dermis and subcutaneous
tissues frequently accompanied by infection. Impetigo is a staphylococcus infection with small macules (unraised spots) or
vesicles (small blisters). Herpes zoster is a viral infection with red papules along the course of a nerve or dermatome.

C160 C4 III
Surgical repair
A stage IV pressure ulcer is characterized by full thickness skin loss with extensive destruction, tissue necrosis, or damage
to muscle, bone or supporting structures. Surgical repair is indicated for patients with extensive and chronic ulcers. The
other interventions are not viable options to resolve this problem.

C161 C6 II
A circumducted gait.
Circumduction is a compensation for weak hip flexors or an inability to shorten the leg (weak knee flexors and ankle
dorsiflexors). Hip hiking can also compensate for an abnormally long leg (lack of knee flexion and dorsiflexion). Excessive
hip flexion is a compensation for foot drop. Forward trunk lean and backward trunk lean are stance phase deviations that
compensate for quadriceps weakness and glutens maximus weakness respectively.

C162 C6 III
Icing and massage to the residual limb
This patient is experiencing phantom pain, a common occurrence seen in an many as 70% of patients. Treatment
interventions can include icing, pulsed ultrasound, TENS, or massage. Medical interventions include injections and
surgical procedures (rhizotomy, neurectomy). Prolonged inactivity and bedrest is contraindicated. Hyaluronidase is
indicated for edema reduction.

C163 C7 III
The validity of the study was threatened with the introduction of sampling bias.
The investigator used a sample of convenience and therefore introduced systematic sampling error (a threat to validity).
Random selection of subjects would improve the validity of this study. Generalization to a group of patients with chronic
ankle sprain cannot be made. The Hawthorn effect refers to the influence the subject’s knowledge of participation in the
experiment had on the results of the study.

C164 C3 III
Left sidelying with the head of the bed in the flat position.
In order to match perfusion and ventilation, the therapist need to place the unaffected side in a gravity dependent position or
that of left sidelying. While prone and supine with the head of the bed in the flat position might helpful, these positions
would not be as likely to show as much improvement as left sidelying, The use of the Trendelenburg position is
inappropriate.

C165 C1 I
A lesion of spinal accessory nerve.
Rotary winging occurs when the inferior angle of one scapula is rotated farther from the spine that the inferior angle of the
other scapula. Although this type of winging could be found with all of the above answers, the shoulder drooping and
inability to shrug the shoulder is secondary to a lesion of the spinal accessory nerve (cranial nerve XI) which innervates the
trapezius muscle.

C166 C5 III
Low detection sensitivity with recording electrodes placed closely together.
By initially placing the electrodes close together, the therapist decreases the likelihood of detecting undesired motor units
from adjacent active muscles (crosstalk). By setting the biofeedback sensitivity (gain) low the therapist would decrease the
amplitude of the signals generated by the hypertonic muscles and keep the EMG output from exceeding a visual and/or
auditory range (scale).

C167 C2 II
Left posterolateral medulla or pons.
A lesion in the posterolateral medulla or lower pons causes mixed sensory loss (described in the question). Pain and
temperature are affected while discriminative touch and proprioception are not (the medial lemniscus is not involved).
Sensory loss will be completely contralateral only after the discriminative sensory tracts are crossed in the upper midbrain.

C168 C2 II
Drowsiness and muscle weakness.
Baclofen, used in the management of spasticity, can produce CNS depression (drowsiness, fatigue, weakness, confusion,
vertigo, dizziness, and insomnia), occurring in less than 10% of patients. Additional adverse effects can include
hypotension and palpitations, and urinary frequency. Vomiting, seizures, and coma are signs of overdosage.
C169 C2 II
Neocerebellum.
The neocerebellum (cerebrocerebellum) controls ipsilateral limb movements, ensuring coordination (adequate force,
direction, extent of movement, ordering and timing). The spinocerebellum and vermis control synergistic action of axial
and girdle muscles (postural stability) while the vestibulocerebellum controls equilibrium responses and head and eye
muscles.
C170 C3 II
Higher heart rate and arterial blood pressure
Dynamic exercise facilitates circulation while isometric (static) exercise hinders blood flow, producing higher heart rates
and arterial blood pressures. The Valsalva maneuver which accompanies breath holding produces increased intrathoracic
pressure, which in turn hinders normal venous return to the heart. Breath holding is more likely with isometric exercise but
is not always present.

C171 C4 III
Ice, elevation and splint for the limb.
Hemarthrosis (bleeding into joint spaces) is associated with swelling, joint pain, and decreased ROM and movement.
Optimal treatment involves pain management along with rest, ice, elevation, and functional splinting. Weightbearing and
exercise during an acute bleed are contraindicated.

C172 C8 II
Refer him to his physician as the therapist suspects a small stroke.
The presence of focal signs (incoordination, anomia) with cognitive signs (memory loss) is indicative of impaired brain
function and may be the result of small strokes. This is the most likely choice given the spotty symptoms he presents with
their sudden onset. Senile dementia, Alzheimer’s type, can include some of the same symptoms but the onset is gradual
and the course is typically slowly progressive. The reporting of these findings to the primary physician should not be
delayed. Further diagnostic workup is indicated. Hospital admission and anesthesia can cause temporary cognitive
difficulties (delirium) but these should not persist with discharge home.

C173 C6 III
Consume a carbohydrate before or during the race to avoid hypoglycemia.
During exercise of increasing intensity and duration, plasma concentrations of insulin progressively decrease. Exercise-
induced hypoglycemia is the likely result. Hypoglycemia can also occur up to 4-6 hours after exercise. To counteract these
effects, the individual may need to reduce his insulin dosage or increase carbohydrate intake before or after running.
Consuming a carbohydrate product before or during the race will have a preventive modulating effect on hypoglycemia.

C174 C6 III
Supracondylar/suprapatellar suspension.
The short amputation limb benefits from supracondylar/suprapatellar (SC/SP) suspension. The medial, anterior and lateral
walls extend above the femoral epicondyles and patella. Patients may experience problems with kneeling with the high
anterior wall. The thigh corset is indicated for individuals with sensitive skin on the amputation limb. Syme’s suspension
is indicated for the patient with ankle disarticulation.

C175 C6 I
Instruct the PTA to have the patient sit down and utilize mental practice of the task.
Mental rehearsal (mental practice) is the best strategy to have the patient learn the correct sequence. In the
nonweightbearing position, the patient’s anxiety is lessened, leaving her free to concentrate on the task at hand. The PT
should provide appropriate guidance to the PTA but not necessarily take over care. Lack of understanding about the gait
sequence rather than balance difficulties seem to be the major problem. Distributed practice with long rest times does not
address the main difficulty.

C176 C1 I
Lateral.
The therapist can observe anterior slippage of one vertebral body on another best using the lateral view. The oblique view
is most effective when evaluating the integrity of the pars interarticularis. The “scotty dog with a collar” indicates that the
pars interarticularis is fractured and spondylolysis, a precursor of spondylolisthesis, is present.

C177 C5 III
Each side allotting five minutes for each section.
The total treatment area is too large for the 5 cm2 sound head to produce adequate tissue heating. Moving the transducer
too fast to cover both sides adequately in the allotted time does not allow sufficient time for the acoustic energy to produce
heat. Sonating the two areas independently will allow more time for the tissue temperature to rise during the treatment time
in each area. Two minutes is too brief to produce sufficient tissue heating.
C178 C5 II
Transcutaneous electrical stimulation
All electrical stimulation devices are contraindicated when a patient has a demand-type pacemaker. The electrical signals
could interfere with the rhythmic signals of the pacemaker.

C179 C2 II
Vestibular deficiency.
The clinical test for sensory integration in balance using dynamic posturography testing is positive for vestibular deficiency
with loss of balance on conditions 5 and 6. Patients who are surface dependent (somatosensory) have difficulties with
conditions 4, 5, and 6. Patients who are visually dependent have difficulties with conditions 2, 3, and 6. Sensory selection
problems are evident with loss of balance on conditions 3-6.

C180 C2 II
Loss of the hippocampus and declarative memory function.
Declarative memory refers to conscious, explicit, or cognitive memory. It is a function of the cerebral cortex, and the
hippocampus. Procedural memory (nonconscious memory or implicit memory) refers to the recall of skills and habits, and
emotional responses. It is the result of integrated action of the frontal cortex (neocortex), thalamus, and striatum of the
basal ganglia.

C181 C3 II
2 daily sessions of 30 minutes at 40-70% VO2 max
This individual is obese (body mass index of 30 kg/m2 or more) and will benefit from exercise to increase energy
expenditure and diet to reduce caloric intake. The initial exercise prescription should utilize low intensity with longer
duration exercise. Splitting the training into 2 sessions each day is a good choice. The goal is to work toward bringing the
target heart rate into a suitable range. Obese individuals are at increased risk of orthopedic injuries and require close
monitoring.

C182 C3 II
Compress with one hand to a depth of one third to one half the depth of the chest at a rate of 100/min.
The correct procedure is to use one hand to compress the infant’s chest to one third or one half its depth. Chest
compression should be at a rate of 100/min. Previous guidelines called for depression of the sternum ½ to 1 inch using 2
fingers. (Source: American Heart Association, Revised CPR Guidelines, December 2005).

C183 C4 II
Hypokalemia.
Hypokalemia, decreased potassium in the blood, is characterized by the above signs and symptoms. Other possible
symptoms include respiratory distress, irritability, confusion or depression, and GI disturbances. Hyperkalemia is excess
potassium in the blood. Hyponatremia is decreased sodium in the blood, and hypocalcemia is decreased calcium in the
blood. These conditions cannot produce this battery of symptoms. Refer to Chapter 6 for signs and symptoms of these
other imbalances.

C184 C8 II
Coronary artery disease.
Lack of oxygen to the brain is the-most probable cause of her fainting. Heart failure with dyspnea and hyperventilation can
decrease cerebral blood flow by as much as 40%. Postprandial hypotension (a drop in BP after a meal) and hypoglycemia
can also cause syncope, but would not likely cause dyspnea and hyperventilation. Seizures would present with additional
clinical features (e.g. an olfactory or visual aura, tongue biting, motor twitching, etc.).

C185 C6 III
Stand in front and slightly to the right side.
The correct guarding position is to stand in front and slightly to the involved side (the light side in this case). During ascent,
the therapist should stand behind and slightly to the involved side.

C186 C6 III
A ramp with a slope of 1:12 with a level landing at the top of at least 60 by 60 inches.
A ramp should have a slope of 1:12 (for every inch of vertical rise, 12 inches of ramp is required). This equals a grade of
8%; a grade of 12% is too steep. A level landing at top of at least 60 by 60 inches is required to allow adequate wheelchair
turning radius. Minimum clearance width for doorways is 32 inches; 36 inches is ideal. A minimum width of 40 inches is
too wide and would require a custom built door. The outside door should open in.
C187 C2 III
Increasing the limits of stability and improving center of pressure alignment.
The patient with Parkinson’s disease exhibits significant balance impairments including loss of postural reflexes, decreased
limits of stability, flexed, stooped posture that alters the center of pressure in an anterior direction, freezing, and orthostatic
hypotension. Platform balance training should work toward improving the limits of stability and center of pressure
alignment (the patient should focus on reducing anterior displacement).

C188 C3 II
Sinus rhythm with upsloping ST segment depression.
The patient is exhibiting sinus rhythm with upsloping ST segment depression. An abnormal ECG response is defined as
greater than or equal to 1.0 mm of horizontal or downsloping depression at 80 msec beyond the J point (Source: American
College of Sports Medicine, ACSM’s Guidelines for Exercise Testing and Prescription, 6th ed., 2000). Associated clinical
signs suggestive of myocardial ischemia include dyspnea and angina. Following a review of this patient’s exercise
responses coupled with the ECG findings, the therapist correctly determines the exercise session does not have to be
terminated. The therapist should continue to closely monitor the patient’s responses.

C189 C8 III
Alternate the side of the wheelchair power control.
Alternating the side of the power control will help keep upper extremity activity and sitting in the wheelchair more
symmetrical. Trunk rotation and extension exercises will not reduce scoliosis. Lateral postural supports may be indicated.
A reclining seat back is not.

C190 C1 II
Facet syndrome.
A facet syndrome presents itself with localized pain. Cervical radiculopathy presents with arm pain in the dermatomal dis-
tribution and increased pain by extension and rotation or side flexion. Cervical strain presents with pain on activity or when
the muscle is on stretch. Cervical disc herniation has a dermatomal pain distribution with an increase of pain on extension,
and pain on flexion may either increase or decrease (most common).

C191 C6 II
Use job rotation during the workday.
Administrative controls reduce the duration, frequency, and severity of exposures to ergonomic stressors. Job rotation
reduces fatigue and stress by rotating the worker to jobs that use different muscle-tendon groups during the workday. The
other choices represent either engineering or educational interventions.

C192 C5 IIII
5-second ramp up, 5-second stimulation, 5-second ramp down.
A relatively long ramp up time over a 5-second period is used to minimize stimulating the muscle too quickly and
increasing the spasticity. The ramp down time has no effect on spasticity.

C193 C2 II
Provide consistent feedback using a blocked practice schedule.
Early learning should focus on consistent feedback given after every trial to improve initial performance. A blocked
practice schedule with repeated practice of the same skill will also reinforce early learning. Variable feedback schedules
(summed or bandwidth) and variable practice schedules (serial and random) are indicated for later learning to improve
retention. Using only guided movement is contraindicated in this case since it minimizes active participation and active
learning.

C194 C2 II
Anterior cerebral artery syndrome.
These clinical manifestations are consistent with anterior cerebral artery syndrome. Patients with middle cerebral artery
syndrome demonstrate the opposite findings, greater involvement of the arm than leg. Patients with posterior cerebral artery
syndrome demonstrate primary involvement of the visual cortex (contralateral homonymous hemianopsia) along with
dyslexia (difficulty reading), prosopagnosia (difficulty naming people on sight), and memory defect (temporal lobe lesion).
Patients with basilar artery syndrome demonstrate a combination of brainstem syndromes along with signs of posterior
cerebral artery syndrome.
C195 C3 I
Elevation-induced pallor and dependent redness with the extremity in the gravity dependent position.
Rubor of dependency test is used to assess the adequacy of arterial circulation by evaluating the skin color changes that
occur with first, extremity elevation (pallor) and then lowering of the extremities (delayed color changes, redness).

C196 C4 III
Pelvic floor and gentle abdominal exercises for the first 4-6 weeks.
Post-cesarean physical therapy can include postoperative TENS, assisted breathing and coughing techniques, and gentle
abdominal exercises with incisional support provided by a pillow. Pelvic floor exercises are also important since hours of
labor and pushing are typically present before surgery. Vigorous exercise is contraindicated for at least 6 weeks.

C197 C8 II
Depression.
Many elderly experience social isolation and major depression. All of the above clinical manifestations can be indicative of
depression. Additional signs and symptoms include: anxiety, crying spells, Jack of self-confidence, low self-esteem and
self-reproach, negative expectations (hopelessness, helplessness, increased dependency); suicidal thoughts and delusions.
While depression can be associated with the other choices, there is no evidence in this case that they are contributing
factors.

C198 C6 III
Adding horizontal grab bars positioned at 45 inches.
Horizontal grab bars should be positioned at an optimal height of 33-36 inches. The toilet seat should be raised to a height
of 17-19 inches. Minimum clearance width for doorways is 32 inches; 36 inches is ideal.

C199 C6 II
Reduce peak rise of the pelvis.
Lateral pelvic tilt in the frontal plane keeps she peak of the sinusoidal curve lower than it would have been if the pelvis did
not drop. Lateral pelvic tilt to the right is controlled by the left hip abductors. Forward and backward rotations of the pelvis
assist the swing leg. The normal physiologic valgum at the knee reduces the width of the base of support. Knee flexion at
midstance is another adjustment in keeping the center of gravity from rising too much. All are termed determinants of gait.

C200 C1 II
Thoracic posture syndrome.
Thoracic posture syndrome presents in people who perform work that involves a lot of sitting in a kyphosed posture for
prolonged periods. A dorsal shift of disc increases the pressure on the posterior longitudinal ligament and the dura mater,
leading to overload of other dorsal ligaments and paravertebral muscles. HNP in the thoracic region is almost always a
result of trauma, and usually involves axial compression (fall on the buttocks) or when carrying a heavy object with the
thoracic spine in flexion. Spondylodiscitis is inflammation of the intervertebral disc that has a hematogenic cause, which
presents with acute or gradual onset of pain in the back with radiation around the side and sometimes even into the legs.
Traumatic compression syndrome is due to an axial force acting on a flexed thoracic spine, most frequently affecting T12,
LI and L2. The patient typically presents with acute immediate pain, with every movement extremely difficult due to pain.

You might also like